You are on page 1of 143

THIN LENSES

1. 1995Q3b P2
An object placed in front of a convex lens of focal length 10 cm produces
an image at a distance of 15 cm from the lens and on the same sides
as the object

Determine the position of the object (4 marks)

2. 1995 Q4 P2
(a) Draw a ray diagram to show how a convex lens works as a
magnifying glass
(5 marks)
(b) The diagram in figure 3 shows a certain eye defect

Figure 3 Eye
Object

Near point
Image

(i) Name the object (1 mark)


(ii) Draw on the same diagram an arrangement to correct
the defect
(1 mark)
3. 1996 Q38 P1
Figure 8 shows an object O placed in front of a concave lens with
principal foci F and F. Construct a ray diagram to locate the position
of the image (3 marks)

Fig. 8

F F

4. 1998 Q6 P1
Figure 2 drawn to scale shows a lens L1 placed 30cm from an
object O. The image is formed on the screen S 50cm from the lens.

Fig. 2

Sketch a diagram to show the position of an object, when a converging


lens is used as a magnifying glass. (3 marks)

1
5. 1998 Q7 P2
a) Fig. 6 shows an object, 0.3cm high placed in front of a concave mirror. C
is the centre of curvature of the mirror. The diagram is drawn to scale:
(1cm: 2cm)

Draw a ray diagram, on figure 6, and determine the size of the image produced.

b) Table 3 shows the object distance y and the corresponding image


distance v, for an object placed in front of a concave mirror.

U(cm) 20 25 30 40 50 70
V(cm) 20 16.7 15 13.3 12.0 11.6
1/v(cm-1)
V(cm-1)
Table 3

i) Complete the table and plot a graph of 1/v (y-axis) against 1/u (give
your answers to 3 decimal places).
ii) From the graph, determine the focal length of the mirror.

6. 1999 Q32 Q1
Sketch a diagram to show the position of an object, when a converging
lens is used as an magnifying glass. (1 mark)

7. 1999 Q2 P2
a) An object O placed in front of a converging lens Lo forms an image 1
on the other side of the lens. Another converging lens LC placed such
that the two lenses form a compound microscope.

i) Draw a diagram of the set up and sketch the rays to show how the
final image is formed. (6 marks)

ii) Give a reason why the focal length of Lo must be greater than that of Lc
(1 mark)

2
(b)
An object is placed 30cm from a converging lens. A focused image is
formed on a screen placed 30cm from the same lens on the other side.
The screen in now moved 5cm towards the lens. Determine the distance
the object must be moved so that a focused image is formed on the screen.
(4 marks)

8. 2000 Q31, 32 P1
31. Fig (19) drawn to scale) shows the image, I, formed by a diverging lens.
F is principal focus of the lens.

F I

FIG. 19

By drawing the appropriate rays on the same diagram, locate the position
of the object. (3 marks)

An armature composed of turns of insulated copper wire wound on


laminated soft – iron core is rotated in a magnetic field to generate
an e.m.f. Use this information to answer questions 31 and 32.

32. State two factors other than the speed of rotation that affect the magnitude
of e.m.f generated

9. 2002 Q28 P1
Fig. 13 represents an object O placed 10cm in front of a diverging lens
is the focal point of the lens.

F
O

Draw rays to locate the position of the image. Determine the image distance.

3
10. 2002 Q30 P1
Fig 21 shows a converging lens whose local point F is marked.

Fig. 21

An object is placed in front of the lens such that the lens forms a real
magnified image.
Sketch on the same diagram array diagram to represent this. (3 marks)

11. 2003 Q16 P1


Figure 10 (draw to scale) shows an image I formed by a diverging lens, L.
L

F I
Fig 10

On the same diagram, draw appropriate rays to locate the position of object.
Determine the object distance. (3 marks)

12. 2003 Q7 P2
a) Fig. 8. shows an experimental set up consisting of a mounted lens. L, a
screen, s, a meter rule and a candle.

Candle L S

Metre rule
Figure 8

(i) Describe how the set-up may be used to determine the focal length,
f, of the lens. (5marks)

(ii) State the reason why the set-up would not work if the lens were
replaced with a diverging lens. (1 mark)

4
(b) The graph in figure 9. Shows the relationship between 1/r and 1/v for
converging lens where u and v are the object and image distances
respectively.

Figure 9

For the graph, determine the focal length, f of lens. (5 marks)

(c) An object placed 15cm from a convex lens is magnified two times.
Determine the focal length of the lens. (4 marks)

13. 2004 Q31 P1


A vertical object is placed at the focal point F of a diverging lens as
shown in Figure 16.

Object

Sketch a ray diagram to show the image of the object. (3 marks)

14. 2005 Q30 P1


An illuminated vertical object is initially placed on the principal axis of a
converging lens and 32cm from it. The focal length of the lens 15cm. The
object is now placed at a point 12cm from the lens and on the same side.
State two changes other than magnification that are observed on the image
formed due to this change.
5
(2 marks)

15. 2005 Q1P2


a) Describe with aid of a labeled diagram an experiment to determine the
focal length of the lens when provided with the following; an illuminated
object, convex lens, a lens, a lens holder, a plane mirror and a metre rule.
(5 marks)
b) A small vertical object is placed 28cm in front of a convex lens of focal
length 12cm. On the grid provided, draw a ray diagram to locate the
image. The lens position is shown.

(Use a scale; 1cm rep re 4cm) (5 marks)


Determine the image distance.

c) Fig 1 shows a human eye with a certain defect

Normal near
point
Eye lens
Figure 1

(i) Name the defect

(ii) On the same diagram, sketch the appropriate lens to correct


the defect and sketch rays to show the effect of the lens. (2 marks)

16. 2006 Q18b P2


A vertical object is placed 20cm in front of a convex lens of focal
length 5 cm
(i) Determine
I. The image distance (3 marks)
II. The magnification (2 marks)
(ii) State two characteristics of the image (2 marks)

17. 2007 Q4 P2
Figure 3 shows an object, O in front of a concave mirror and its image,
I formed after reflection.

6
(a) On the same diagram draw appropriate ray (s) to locate the
principal focus, F, of the mirror. (2 marks)

(b) Determine the focal length of the mirror (scale 1: 5) (1 mark)

18. 2007 Q10 P2


State what is meant by the term accommodation as applied to the human eye.
(1 mark)
19. 2007 Q16 P2
(a) Figures 11 (a) and (b) show diagrams of the human eye

Fig 11

(i) Sketch in figure 11 (a) a ray diagram to show short-sightedness (1 mark)

(ii) Sketch in figure 11 (b) a ray diagram to show how a lens can be used to
correct the short-sightedness (2 marks)

(b) Figure 12 shows the features of a simple camera

Focusing Spool
ring
Lens

A B

Shutter

Figure 12

(i) Name the parts labeled A and B (2 marks)

(ii) A still object is placed at a certain distance from the camera.


Explain the adjustments necessary for a clear image of the
object to be formed.
(iii) State the functions of the shutter and the parts labeled A and B
(3 marks)
(c) A lens forms a clear image on a screen when the distance between the

7
screen and the object is 80cm. If the image is 3 times the height of the
object, determine.
(i) The distance of the image from the lens (3 marks)
(ii) The focal length of the lens (2 marks)

20. 2008 Q5 P2
The diagram in figure 2 shows an object O placed in front of a converging
lens. F and F are the principal foci for the lens.

Lens

F F

Figure 2

The object is now moved along the principal axis until a virtual image is produced.
On the same diagram:
(i) Draw the object O in the new position along the principal axis;
(ii) Sketch rays to show formation of the virtual image (1 mark)

21. 2008 Q11 P2


Figure 7 shows how rays from a distant and a near object are
focused inside a human eye with a certain defect.

Rays from
different object
Figure 7

Rays from
near object
Figure 7
Name the defect and state the cause of this defect. (1 mark)

22. 2009 Q9 P2
In an experiment, pin, converging lens and a plain mirror are arranged
as shown in figure 4. The distance between the pin and the plain mirror
is l cm while the distance between the lens and the plain mirror is q cm.
The position of the pin is adjusted until its tip coincides with its real image.

image Converging lens


Plane mirror
pin

8
q figure 4
L

State the focal length of the lens (1 mark)

23. 2010 Q16 P2


(a) Figure 10, shows an object placed in front of a converging lens of
focal length 50mm.

On the same figure, draw a ray diagram showing the location of the image. (3 marks)

i) Use the diagram to determine the:


(I) Image distance. (1 mark)
(II) Magnification. (2 marks)

(ii) State the adjustment that should be done to obtain a larger virtual
image using the same lens. (1 mark)

(iv) State one application of the arrangement in Figure 10. (1 mark)

(b) Figure 11, shows a pin 60mm long placed along the principal axis of the
lens used in part (a). The near end of the pin is 80 mm from the lens.

Pin Principal axis

Figure 11

Determine the length of the image (5 marks)

24. 2011 Q16 P2


(a) State the meaning of the term ″principal focus″ as applied in lenses.

(1 mark)

9
(b) You are provided with the following apparatus to determine the
focal length of a lense:
 a biconvex lens and a lens holder
 a lit candle
 a white screen
 a meter rule

(i) Draw a diagram to show how you would arrange the above
apparatus to determine the focal length of the lenses (1 mark)

(ii) Describe the procedure you would follow (1 mark)


(iii) State two measurements you would take. (2 marks)

(iv) Explain how the measurements in (iii) would be used to


determine the focal length. (2 marks)

(c) an object is placed 30cm infront of a concave lens of focal length


20cm. Determine the magnification of the image produced. (4 marks)

25. 2012 Q9 P2
Figure 8, shows an object O placed in front of a diverging lens whose
principal focus is F.
On the figure, draw a ray diagram to locate the image formed (3 marks)

26. 2012 Q17 P2


a) Figure 16, shows a graph of magnification against object distance, for
an object placed in front of a lens of a focal length 20cm.
Using the graph

Magnification

10
Figure 16

Object distance (cm)

i. State the effect on the size of the image when the object distance is
increased from 25cm (1mark)

ii. Determine the distance between the object and the lens when the
image is the same size as the object (2marks)

iii. Determine the image distance when the object distance is 25cm (3marks)

27. 2013 Q14 P2


Figure 6 shows two convex lenses A and B used to produce a magnified
virtual image of an object.

Final image

Figure 6

(a) Determine the focal length of lens A. (Take 1 unit to represent 10cm) (1mark)

11
(b) State the function of:
(i) Lens A (1mark)
(ii) Lens B (1mark)

(c) State how the functions in (b) are achieved by:


(i) Lens A (1mark)
(ii) Lens B (1mark)

(d) Determine the magnification produced by:


(i) Lens A (2marks)
(ii) The whole system (2marks)

28. 2014 Q1 P2
Figure 1 shows two parallel rays from a distant object passing through
a convex lens:

Figure 1

a) Indicate on the diagram, the position of the principal focus of the lens (1 mark)
b) Determine the focal length of the lens (1 mark)

29. 2015 Q18b P2


A real object of height 1 cm placed 50 mm from a converging lens
forms a virtual image 100 mm from the lens.
(i) Determine the:
(I) focal length of the lens; (3 marks)
(II) magnification (2 marks)

ii) On the grid provided draw to scale the ray diagram for the set up, to
show how the image is formed (3 marks)

12
UNIFORM CIRCULAR MOTION
1. 1995 Q4c P2
(i) Explain why a pail of water can be swung in vertical circle without
the water pouring out (3 marks)

(ii) A car of mass 1200kg is moving with a velocity of 25ms-1 around a


flat bend of radius 150m. Determine the minimum frictional force
between the tyres and the road that will prevent the car from sliding off.
(4 marks)
2. 1996 Q6 P2
(a) The fig. 4 shows the diagram of set up to investigate the variation
of centripetal with the radius r, of the circle in which a body rotated

Body
Smooth
rotating table
String

Hanging masses
Fig. 4

Describe how the set up can be used to carry out the investigation (5 marks)

(b) Table 1
Mass, m (g)
60 50 40 30 20

Radius, r (cm) 50 41 33 24 16

13
Table 1 shows results obtained from an investigation similar to the one in part (a)
(i) Plot a graph of force, F ( y- axis) on the body against the radius ,
r, ( in meters) (5marks)
(ii) Given that the mass of the body is 100g, use the graph to
determine the angular velocity, (5 marks)

3. 1998 Q18 P1
State the reason why the amplitude of a simple pendulum decreases with time.

4. 1998 Q29 P1
Fig. 8 shows a car of mass M moving along a curved part of the road with a
constant speed.

Explain the fact that the car is more likely to slide at B than at A if the
speed is not changed.

5. 1999 Q31 P1
Figure 15 shows two masses 0.1kg and 0.2kg connected by a string
through a hole on a smooth horizontal surface.

0.2 kg Fig. 15

The 0.1kg mass rotates in a horizontal circle of radius 3cm. Calculate the
angular velocity of the mass when the system is in equilibrium. Use
acceleration due to gravity g= 10ms-2 (3 marks)

6. 2000 Q5 P2
(a) Fig 8 shows a container with small holes at the bottom in which wet
clothes have been put. When the container is whirled in air at high speed
as shown, it is observed that the clothes dry faster.

Container Rotation

Holes

Wet cloth

14
Explain how the rotation of the container causes the clothes to dry faster.
(3 marks)

(b) (i) A glass block of mass 100g is placed in turn at various distances from
the centre of a table which is rotating at constant angular velocity. It is
found that a distance of 8.0 cm from the centre, the block just starts to
slide off the table. If the force of the friction between the block and the
table is 0.4 N determine.

(I) The angular velocity of the table (3 marks)


(II) The force required to hold the block at a distance of 12 cm from the
centre of the table. (3 marks)

(ii) A glass of mass 200 g is now placed at a distance of 8.0 cm from the
centre of the table in (i) above, and the table rotated at the same
constant angular velocity. State with a reason whether or not the
block will slide. (3 marks)

7. 2001 Q29 P1
Fig. 20 shows a ball being whirled in a vertical plane.

Ball

String

Fig 2A

Sketch on the same figure the path followed by the ball if the string cuts
when the ball is at the position shown in the figure. (1 mark)

8. 2002 Q26 P1
A small object moving in a horizontal circle of radius 0.2m makes 8
revolutions per second. Determine its centripetal acceleration. (3 marks)

9. 2002 Q2b P2
A child of mass 20kg sits on a swing of length 4m and swings through a
vertical height of 0.9m as shown in figure 2..

Determine:
15
(i) Speed of the child when passing through the lowest point. (3marks)
(ii) Force exerted on the child by the seat of swing when passing through
the lowest point. (3marks)

10. 2004 Q29 P1


A body mass 0.50kg is attached to the end of a string of length 50cm and
whirled in a horizontal circle. If the tension in the string is 81N, determine
the velocity of the body. (3 marks)

11. 2005 Q29 P1


A car of mass 800kg moves on a circular track of radius 20m. The force of
friction between the tyres and the tarmac is 4800N. Determine the maximum
speed at which the car can be driven on the track without skidding.

12. 2005 Q6 P2
(a) Define the term angular velocity (1 mark)

(b) A body moving with uniform angular velocity is found to have covered
an angular distance of 170 radians in t seconds. Thirteen seconds later
it is found to have covered a total angular distance of 300 radians.
Determine t. (3 marks)

(c) Fig 8 shows a body of mass m attached to the centre of a rotating table
with a string whose tension can be measured. (The device for measuring
the tension is not shown in the figure)

Figure 8
String
The tension, T on the string was measured for various values of angular
velocity, w. The distance r of the body from the centre was maintained
at 30cm. Table 1 shows the results obtained.

Table 1
W2 4.0 9.0 16.0 25.0 36.0
Angular velocity w (radi-1) 2.0 3.0 4.0 5.0 6.0
Tension T (N) 0.04 0.34 0.76 1.30 1.96

(i) Plot the graph of T (y- axis against w2) (5 marks)


(ii) From the graph, determine the mass, m of the body given that
T = mw2 –C Where C is a constant (4 marks)

(iii) Determine the constant C and suggest what it represents in the set up.
(2 marks)
13. 2006 Q10 P1
16
Figure 6 shows the path taken by a matatu traveling on a horizontal ground
(a winding road)
D F
B

C
A G
E
Figure 6

The speed of the matatu is constant. Identify with reason the point along
the path which a load placed loosely on the rack (carrier) of the matatu is
most likely to roll off. (2 marks)

14. 2007 Q18 P1


(a) State what is meant by centripetal acceleration
(b) Figure 12 shows masses, A, B and C placed at different points on a rotating
table. The angular velocity, , of the table can be varied.

Rotating table

Masses
Fig 12

(i) State two factors that determine whether a particular mass


slides off the table or not (2 marks)

(ii) It is found that the masses slide off at angular velocities A, B, and C
respectively. Arrange the values of A, B, C in decreasing order.
(1 mark)
(c) A block of mass 200g is placed on a frictionless rotating table
while fixed to the centre of the table by a thin thread. The distance
from the centre of the table to the block is 15 cm. If the maximum
tension the thread can withstand is 5.6N. Determine the maximum
angular velocity the table can attain before the thread cuts. (4 marks)

15. 2009 Q14 P1


A turntable of radius 8cm is rotating at 33 revolutions per second.
Determine the inear speed of a point on the circumference of the turntable. (3 marks)

17
16. 2009 Q19 P1
(a) Define angular velocity. (1 mark)

(b) Three masses are placed on a rotating table at distances 6cm, 9cm and
12cm respectively from the centre of rotation. When the frequency of
rotation is varied, it is noted that each mass slides off at a different
frequency of rotation of the table. Table 1 shows the frequency at
which each mass slides off.

Table 1
Radius r (cm) 12 9 6
Sliding off 0.68 0.78 1.0
Frequency, f, (rev/s)

(i) State two factors that determine the frequency at which each mass
slides off (2 marks)

(ii)Oil is now poured on the table before placing the masses .Explain the
Effect of this on the frequency at which each mass slides off.. (2 marks)

(c) Figure 12 shows a flywheel of a radius 14cm suspended about a


horizontal axis through its centre so that it can rotate freely about
the axis. A thread is wrapped round the wheel and a mass attached
to its loose end so as to hang at a point 1.26m above the ground.

14cm Flywheel

Thread

Mass

1.26m

Figure 12.

When the mass is released, it accelerates at 0.28ms-2. Determine the angular


velocity of the wheel just before the mass strikes the ground. (4 marks).

17. 2010 Q12 P1


Figure 5 shows a mass of 200g connected by a string through a hollow tube
18
to a mass of 0.5kg. Teh 0.5kg mass is kept stationary in the air by whirling the
200g mass round in a horizontal circle of radius 1.0 metre.

Mass = 200g

Hollow tube

String

Mass = 0.5kg
Figure 5

Determine the angular velocity of the 200g mass. (3 marks)

18. 2011 Q12 P1


Figure 11 shows a stone attached to the end of a string moving in a
horizontal circle with a uniform speed of 2ms"1. When the stone reaches
point X on the circle, the string breaks.

X String

Stone

(i) Indicate on the diagram with an arrow, the direction of the motion
of the stone (1 mark)

(ii) when the string breaks. (1 mark)


(iii)State the magnitude of the velocity after the string breaks. (1 mark)
(ii) Give a reason for your answers in (i) and (ii) (1 mark)

19. 2013 Q16c P1


Figure 10 shows the path of an object of mass m attached to a string of
length r when whirled in a vertical circle at a constant speed V, a is the
highest point on its path

19
Object
String

Motion

Figure 10

(i) State the forces that provide the centripetal force on the object when
it is at point A. (2 marks)
(ii) Indicate with an arrow on the diagram the direction of the net force F
acting on the object when it is at A. (1 mark)

20. 2013 Q14 (a) & (b) P1


(a) State two ways in which the centripetal force on a body of mass M can
be increased. (2 marks)

(b) Figure 7 shows an object at the end of a light spring balance connected
to a peg using a string.
The object is moving in a circular path on a smooth horizontal table with a
constant speed.

Spring balance

Object peg

Figure 7

(i) State what provides the centripetal force. (1 mark)


(ii) Indicate with an arrow on the figure the direction of centripetal force.
(1 mark)
(iii) State a reason why the object is accelerating while its speed remains
constant. (1 mark)
(iv) Given that the mass of the object is 0.5 Kg and it is moving at a speed of 8
ms-1 at a radius of 2m, determine the reading on the spring balance.
(3 marks)

21. 2014 Q18 P1


(a)When a bus goes round a bend on a flat road, it experiences a centripetal force.
State what provides the centripetal force. (1 mark)

(b) State the purpose of banking roads at bends (1 mark)

(c) A student whirls a stone of mass 0.2 kg tied to a string of length 0.4 m in a
vertical plane at a constant speed of 2 revolutions per second. (Take
acceleration due to gravity gas 10ms-2)

20
(i) State two forces acting on the stone when it is at the highest point (2
marks)

(ii) Determine the:

(I) Angular velocity of the stone (3 marks)

(II) Tension in the string when the stone is at the highest point (3 marks)

22. 2015 Q10d P1


Figure 12 shows the path of alight ball projected horizontally.

The ball is then made to spin in an anticlockwise direction as it moves:


(i) on the same axis, sketch the new path of the ball. (1 mark)
(ii) explain how the ball attains the new path. (2 marks)

23. 2016 Q17 P1


(a) When a bus goes round a bend on a flat road, it experiences a centripetal
force. State what provides the centripetal force. (1 mark)

(b) State the purpose of banking roads at bends. . (2 marks)

c) a student whirls a stone of mass 0.2 kg tied to string of length 0.4 m in a


vertical plane t a constant speed of 2 revolutions per se cond. (Take
acceleration due to gravity g = 10ms -2)

i) State two forces acting on the stone when it is at the highest point (2 mks)
ii)Determine the
i. Angular velocity of the stone (3mks)
ii. Tension in the string when the stone is at the highest point (3mk)

21
FLOATING AND SINKING
1. 1994 Q5a P2
(a) State Archimedes’s principal (1 mark)

2. 1996 Q29 P1
A solid copper sphere will sink in water while a hollow copper sphere
of the same mass many float. Explain this observation (2 marks)

3. 1998 Q2b P2
Fig. 1 shows a block with a graduated side, and of dimension 4cm x-4cm x 4cm x
16cm, just about to be lowered into a liquid contained in an overflow can.
Thread
16mm long
block

22
Overflow

Can

Balance Fig 1

During an experiment with this set-up, the following information


was recorded;
-The block floated with three quarters of it submerged
-Initial reading of balance=0 grammes
-final reading of balance= 154 grammes.
Use the information to determine the density of the:
(i) Block (ii) Liquid
(Use g= 10ms2 . give your answers to 1 decimal place.)

4. 1999 Q29 P1
A concrete block of volume V is totally immersed in seawater of density p.
Write an expression for the up thrust on the block. (1 mark)

5. 1999 Q6 P2
a) Explain the following observations: ice cube float on water and
solid benzene sinks in liquid benzene. (2 marks)

b) (i) You are provided with the following:


-An overflow can - A beaker -A spring balance
- A metal block -Water and - String
Describe an experiment to verify Archimedes principle. (6 marks)

ii) A block of wood weighing 2.0N is held under water by a string


attached to the bottom of a container. The tension in the string is
0.5N. Determine the density of the wood. (4 marks)

6. 2000 Q4b P2
Fig 7 represents a block of uniform cross sectional area of 6.0cm 2 floating
on two liquids A and B. The lengths of the block in each liquid are shown.

Figure 7

Given that the density of liquid A is 800kgm-3 and that of liquid B


23
is 1000kgm3 determine the:
(i) weight of liquid A displaced (3 marks)
(ii) Weight of liquid B displaced (2 marks)
(iii) Density of the block (4 marks)

7. 2001 Q2 P1
Fig 2 shows a uniform bar in equilibrium.

A B

Figure 2

When water is added into the beakers A and B until the weights are
submerged, it is observed that the bar tips towards B. Explain this
observation. (2 marks)

8. 2001 Q22 P1
Fig. 15 shows a tall jar containing two fluids A and B. The viscosity of A
is higher than that of B. A solid sphere is released at the top of the jar
and falls through the fluids.

Sphere V

Fig. 15
t
On the axes provided, sketch the velocity – time graph for the motion of
the spheres through the fluids.
9. 2001 Q28 P1
Fig. 19 shows two spheres made of wax each of mass 0.10kg held in
a liquid by strings A and B.

Liquid

Wax spheres
Fig 19

If the upthrust on each sphere is 1.05N, determine the tension in each string.
(acceleration due to gravity g = 10ms-2) (3 marks)

10. 2002 Q25 P1

24
A block of glass of mass 250g floats in mercury. What volume of glass
lies under the surface of the mercury? (Density of mercury is
13.6 x 103 kgm-3). (3 marks)

11. 2003 Q20 P1


When a piece of metal is placed on water, it sinks. But when the same
piece of metal is placed on a block of wood, both are found to float.
Explain this observation. (2 marks)

12. 2004 Q2 P1
The system in figure 2 is in equilibrium at room temperature.
The system is taken outside where the temperature is 100c higher for sometime.

Light
bar
Ballon Small
mass
Air

Fig 2

Explain why it tips to the right immediately it is returned to the room.

13. 2004 Q1 P2
a) A test tube of uniform cross-section loaded so that it can float upright in
(2 marks)
Water. With the aid of a labeled diagram, describe how the test tube
may be calibrated to measure the density of liquid. (4 marks)

b) In an experiment to determine the density of a liquid, a uniform


metal cylinder of cross-section area 6.2cm was hang from a spring
balance and lowered gradually into the liquid. The up thrust was
determined for various submerged lengths. The results obtained
are shown on the graph in Fig 1.

25
Upthrust (N)

Figure 1

SUBMERGED LENTH (cm)

Figure 1
Using graph, determine;
(i) The value of the up thrust when the cylinder is fully submerged (2 marks)
(ii) The Density of the liquid (2 marks)

14. 2005 Q3 P1
The light uniform bar in Fig 3 is equilibrium. The two beakers A and
B contain water at the same temperature. The two blocks are made
of the same material.

Water
Water

A
B
Figure 3
If the temperature of the water in beaker A is now raised, explain
why the beam tips to side A. (4 marks)
Assume the solid does not expand. (4 marks)

15. 2006 Q12 P1


The uniform bar in figure 7 is provided at its midpoint. It is in equilibrium
under the action of two identical balloons filled with equal volumes of

26
different light gases at the same temperature.
Ballons

Figure 7

Uniform
bat

Explain why the bar may not remain in equilibrium if the temperature
of the surrounding changes. (2 marks)

16. 2006 Q17 P1


(a) state Archimedes principle (4 marks)

(b) In an experiment to determine the relative density of methylated


spirit applying Archimedes Principal, the following were provided, a
spring balance, some masses, a piece of thread, water in a beaker
and methylated spirit in a beaker.

The table below shows the results obtained.

Mass (g) 100 150 200


Weight in air (N) 1.00 1.50 2.00
Weight in water (N) 0.88 1.32 1.76

Weight in spirit (N) 0.91 1.36 1.82

(i) Draw labeled sketch diagrams to show how the readings in the
table were obtained (1 mark)
(ii) For each mass, determine the upthrust in water and the upthrust
in the Spirit (2 marks)
(iii) Determine the average relative density of the spirit (3 marks)

(c) A weather balloon of volume 1.2m3 is tied to a rigid support while


being filled with helium gas. The mass of the fabric making the
balloon is 0.30kg.
Determine the maximum tension on the string trying the balloon
to the rigid support (4 marks)

17. 2007 Q19 P1


(a) State the law of floatation (1 mark)
(b) Figure 13 shows a simple hydrometer (4 marks)

27
Uniform tube

Glass club

Lead shots
Figure 13
(i) State the purpose of the lead shots in the glass bulb (1 mark)
(ii)How would the hydrometer be made more sensitive? (1 mark)
(iii) Describe how the hydrometer is calibrated to measure relative density

(c) Figure 14 shows a cork floating on water and held to the bottom of
the beaker by a thin thread.

Cork

Thread
Water

Figure 14

(i) Name the forces acting on the cork (3 marks)


(ii) Describe how each of the forces mentioned in (i) above
changes when water is added into the beaker until it fills up.(3 marks)

18. 2008 Q14 P1


The system in Fig. 8 is in equilibrium
Metre
rule
Pivot

Water

Figure 8

When the temperature of the water is raised the system is observed to


tilt to the right, state the reason for this observation. (2marks)

19. 2009 Q17 P1


(a) State the law of flotation (1 mark)

28
(b) Figure 10 shows a rectangular metal block of density 10500 kgm 3
and dimensions 30cm x 20cm x 20cm suspended inside a liquid
of density 1200kgm 3 by a string attached to a point above the liquid.
The three forces acting on the block are; the tension T, on the string,
weight W of the block, and the upthrust, U, due to the liquid.

String

Liquid

Block

Figure 10

(i) Write the expression relating T, W and U when the block is in


equilibrium inside the liquid. (1 mark)
(ii) Determine the weight, W, of the block. (3 marks)
(iii) Determine the weight of the liquid displaced by the fully
Submerged block. (2 marks)
(iv) Hence determine the tension, T, in the string. (1 mark)

(c) A certain solid of volume 50cm3 displaces 10cm3 of kerosene


(density 800 kgm3) when floating. Determine the density of the solid. (4 marks)

20. 2010 Q10 P1


The weight of a solid in air is 5.0N. when it is fully immersed in a liquid
of density 800kgm-3 its weight is 4.04N. Determine:

a) The upthrust in the liquid (1 mark)

b) The volume of the solid. (2 marks)

21. 2010 Q18 P1


Figure 8 shows a stone of mass 4.0kg immersed in water and suspended
from a spring balanced with a string. The beaker was placed on a
compression balance whose reading was 85N. The density of the stone
was 3000kg-3 while the density of the liquid was 800kg-3.

29
Spring balance

Stone

Water

Reading 85N
Compression Balance

Figure 8
Determine the:
a) Volume of the liquid displaced. (2 marks)
b) Upthrust on the tone (4 marks)
c) Reading of the spring balance: (2 marks)
d) Reading of the compression balance when the stone was
removed from the water. (2 marks)

22. 2011 Q19a, b, e P1

(a) State the conditions necessary for a body to float in a fluid. (1 mark)

(b) A ship made of steel is observed to float on water yet the density
of steel is approximately eight times that of water. Explain this
observation. (2 marks)
(c)
Figure 17 shows three stages of an experiment to determine
relative density of cork which normally floats on water. To make
a sink, a sinker is hang below the cork.

Spring balance

String String

Cork
Cork
Figure 17
Sticker

Water

In (I) a spring balance is used to measure the weight W of the cork


in air. In (II) the spring balance is used to measure the apparent
weight W when only the sinker is submerged in water.
30
In (III) the spring balance is used to measure the apparent weight W 2 when
both cork and sinker are submerged.

(c) Figure 18 shows parts of a simple submarine, a ship that can travel
both on water and under water.
To do this water is pumped in or out of the ballast tanks.

Ballast tank Ballast tank

Pump Figure 18

Explain how the tanks are used to change the depth of the submarine. (2 marks)

23. 2012 Q7 P1
A balloon is filled with a gas which is lighter than air. It is observed
to rise in air upto a certain height. State a reason why the balloon
stops rising. (1mark)

24. 2012 Q6 P1
State two environmental hazards that may occur when oil spills over
a large surface area of the sea. (2 marks)

25. 2012 Q18b P1


Figure 13 shows a log of wood of mass 20kg submerged in water in a
pond and held in position by striking fixed to the bottom of the pond.

Given that density of water is 1000kgm -3 and that of wood is 800kg-3,


determine the;
(i) Volume of the log, (3 marks)
(ii) Upthrust on the log. (2 marks)
(iii)Tension in the string (2 marks)

31
26. 2013 Q18b P1
(a) Figure 12 shows a weighing balance on which a beaker containing
some water is placed. The reading on the balance is 2.80 N. A metal block
weighing 2.7 N is suspended from a spring balance.

Spring

Metal block

Water

Weighing balance

(i) State what is observed on the spring balance and the weighing balance,
as the metal block is gradually lowered into the water.

(I) Observation on spring balance. (1 mark)

(II) Observation on weighing balance. (1 mark)

(ii) Explain the observation made on the spring balance in (I). (2marks)

(iii) When the metal block is fully immersed in the water, the reading on
the spring balnce is found to be 2.46 N. Determine the:

(I) reading on the weighing balance. (2 marks)

(II) density of the metal. (3 marks)

32
(c) Figure 13 shows a hydrometer with a thin stem floating in water
in a beaker.

Water

Lead shot

Figure 13
State with a reason what is observed on the hydrometer when the
temperature of the water is raised. (2 marks)

27. 2014 Q19 P1


Figure 11 shows a test tube whose cross sectional area is 2cm 2 partially
filled with lead shot floating vertically in water.
(Take gravitational acceleration as 10ms2 and density of water pws 1 g cm-3)

(a) (i) Determine the:


(I) Volume of the water displaced (2 marks)
(II) Weight of water displaced (3 marks)

(ii) State the combined weight of the test tube and the lead shot (1 mark)

(iii) Determine the length of the test-tube that would be submerged


in a liquid of density 0.8 g cm-3 (4 marks)

(b) The set up in figure 11 can be used as a hydrometer to measure


densities of liquids.
State how such a hydrometer would be improved to measure
small differences in densities of liquids. (1 mark)

33
28. 2015 Q18b P1
Figure 14 shows a block floating in water

When the water is heated; it is observed that he block sinks further.


Explain this observation (2 marks)

34
ELECTROMAGNETIC SPECTRUM
1. 1995 Q26 P1
The table below shows the type of radiation, detection methods and uses of
electromagnetic radiations. Complete the table.

Type of radiation Detector Uses


Ultra violet Photographic paper fluorescence
material

Phototransistor blackened Warmth sensation


thermometer

Radio waves Communication

2. 1996 Q2 P1
What measurable quality is associated with colors of light? (1 mark)

3. 1998 Q20 P1
Table 1 carries information on the type of radiation, detector and use
for some of the electromagnetic radiations. Fill in the blanks.

Type of radiation Detector Used


Microwave Microwave receiver
Visible light Seeing / vision

4. 1999 Q28 P1
Arrange the following in order of increasing frequencies –Gamma
radiation, radio waves, infrared, and X –rays. (1 mark)

5. 2000 Q29 P1
State two uses of microwaves. (2 marks)

6. 2000 Q34 P1
An atom changes from an excited state to an unexcited state releasing
energy. State one factor that affects the frequency of the radiation
released. (1 mark)

7. 2001 Q27 P1
State the reason why radio waves signals are easier to receive than
TV (television) signals in a place surrounded by hills. (1 mark)

8. 2001 Q4b P2
In an excited hydrogen atom. An electron moves from an energy level
Of -1.36 x 10 -19J. Determine the wavelength of the radiation emitted.
(Planks constant h= 6.63 x 10-34 Js and speed of light c= 3.0 x 108 ms-1).

35
9. 2002 Q16 P1
The chart below shows an arrangement of different parts of the
electromagnetic spectrum.

RADIO INFRARED VISIBLE A X - RAYS GAMMA RAYS

Name the radiation represented by A. (1 mark)

10. 2003 Q31 P1


Arrange the following in order of increasing frequency:
Visible light, infrared radiation, X – rays, u. v. radiation, radio waves. (1 mark)

11. 2004 Q28 P1


State the difference between X-rays and Gamma rays in the way in
which they are produced. (1 mark)

12. 2005 Q31 P1


Explain how an “excited’ hydrogen atom is able to emit radiations
of different wavelengths. (1 mark)

13. 2011 Q12 P2


Table 1 shows radiations and their respective frequencies.

Table 1

Type of Yellow Gamma rays Radio waves Micro waves


radiation light
Frequency (Hz) 1×1015 1×1022 1× 106 1× 1011

Arrange the radiations in order of increasing energy. (1 mark)

14. 2013 Q11 P2


Explain the fact that radiant heat from the sun penetrates a glass sheet
while radiant heat from burning wood is cut off by the glass sheet. (2 marks)

15. 2014 Q4 P2
The frequency of an electromagnetic wave is 4.0 X 106 Hz. determine
its wavelength. (take speed of light as 3.0 x 108 ms-1). (3 marks)

36
ELECTROMAGNETIC INDUCTION
1. 1995 Q5 P2
(a) (i) State the law of electromagnetic induction (2 marks)
(ii) Describe an experiment to demonstrate Faraday’s law (4 marks)

(b) (i) A researcher studying the behaviour of step- up transformer


made the following observations:

“More joules per coulomb and fewer coulombs per second at the output
than at the input terminals

Explain why the observation does not imply a violation of the principle
of conservation of energy (4 marks)

(ii) A transformer of 480 turns in the primary coil is used to connect a


9 volt a.c electric device to a 240 v.a.c mains power supply. Calculate
the number of turns in the secondary coil. (3 marks)

2. 1996 Q 31 P1
What causes electromagnetic damping in a moving coil galvanometer (1 mark)

3. 1997 Q19 P1
An object dropped from a height h attains a velocity of 6 ms-1 just
before hitting the ground. Find the value of h. (3 marks)

4. 1997 Q19 P1
State how eddy currents are reduced in a transformer (1 mark)

5. 1998 Q8 P1
The primary coils of a transformer has 2000 turns and carries a current
of 3A. If the secondary coil is designed to carry a current of 30A,
calculate the maximum number of turns in the secondary coil. (3 marks)

6. 1998 Q13 P1
State the reason why a voltmeter of high resistance is more accurate
in measuring potential differences, that one of low resistance.

7. 2000 Q32 P1
State tow factors other than the speed of rotation that affect the
molecule of the e.m.f generated. (2 marks)

8. 2000 Q33 P1
State the reason why soft iron is laminated. (2 marks)

9. 2000 Q4a P2
(i) State one property of soft iron that makes it suitable for use as a
transformer core.

37
(ii) Fig 6 represents a step- down transformer with 500 turns in
the primary and 50 turns in the secondary. The turns are wound
uniformly on the core. The lengths of PQ and QR are indicated.
Determine the p.d across PQ.

Figure 1
10. 2001 Q31 P1
Fig. 22 shows an electric generator. The points P and Q are connected to
a cathode ray oscilloscope (CRO).

Coil rotation

Voltage

P Cummulator

Time (t)
Figure 22
Sketch on the axes provided the graph of the voltage output as seen
on the CRO Given that when t = 0 the coil is at the position shown
in the figure.

11. 2002 Q33 P1


Fig. 15 shows a wire XY placed in a magnetic field.

38
Galvanator

State the direction in which the wire must be moved for the current to
move in the direction shown. (1 mark)

12. 2003 Q32 P1


Two identical copper coils p and Q are placed close to each other as
shown in Figure 17. Coil P is connected to a D>C power supply and
coil Q is connected to a galvanometer, G.

Use this information to answer questions 12 and 13.


State and explain what would be observed on the galvanometer
immediately the switch S is closed.

13. 2003 Q33 P1


State with reason the difference that would be noted in the observation
made in question 32 if the number of turns in coils Q were doubled.

14. 2004 Q5 P2
a) Fig 6 shows a simple generator. The coils are rotated in the anticlockwise
direction as shown.

Axis of rotation
Coil
Figure 6

Slip rings
Brushes

i) Indicate using an arrow on the figure, the direction of the induced


current as the coil passes the position shown.

ii) State two ways of increasing the magnitude of the induced current
39
in this type of generator.

iii) On the axes provided, sketch the graph of the induced e.m.f with time.

Induced
e.m.f

Time (t)

iv) The section marked XY is cut off and a diode inserted. On the axes
provided, sketch the graph of p.d across the resistor R, against time.

p.d (v)

Time (t)

b) Fig 7 shows pendulum A and pendulum B freely suspended between


the poles of identical magnets. Pendulum a is made of thick copper
plate while B is made a copper plate with slots

B
pendulum
Thick copper
Copper plate
plate
with slots

Pole pieces
Fig 7

When the two are set to swing, it is observed that A slows down faster

40
than B Explain this observation. (1 mark)

c) An alternating current source has a root-mean-square potential


difference of 12, V; Determine the peak value of this potential difference.
(4 marks)

15. 2005 Q12, 13 P1


Fig 9 shows a current carrying coil in a magnetic field. The direction
of the current and the resulting force are shown. Study the figure
and answer questions 12 and 13.

F
Fig 9

Split ring commuter

12. Label the poles of the magnets. (1 mark)


13. Explain the purpose of the split ring commutator in the principle
of the D.C motor shown in the diagram. (2 marks)

16. 2005 Q39 P1


In the setup in Figure 19, the magnet is moved towards the coil and
stopped when inside the coil.

Figure 19

It is observed that the galvanometer deflects to side and then goes


back to zero. Explain this observation. (2 marks)

17. 2006 Q20 P1


Figure 10 shows a simple electric generator

Coil Rotation

41
N

P
Alternating
e.m.f
Q Figure 10
(a) (i) Name the parts labeled P and Q (2 marks)
(ii) The e.m.f generated as the coil rotates is represented in the
graph in figure

Emf (v)

Angle (o)

Figure 11

Give reasons for the changes in emf as the coil rotates from 0 0 to
900 and 900 1800 (3 marks)
(a) The primary coil of a transformer has 1200 turns and the secondary
coil has 60 turns. The transformer is connected to a 240V. a.c source.

Determine
(i) The output voltage (2 marks)

(ii) The output current when the primary coil has a current of 0.5A.
(Assume there are no energy losses.) (3 marks)

18. 2007 Q17 P2


(a) State Lenz’s Law of electromagnetic induction (1 mark)

(b) Figure 13 shows a simple microphone in which sound waves from


the person talking cause the cardboard diaphragm to vibrate

Cardboard
diaphram

To amplifier

Sound waves
Coil
Magnet

Figure 13

42
(i) Explain how a varying current is induced in the coil when
the diaphragm vibrates (3 marks)

(ii) State two ways in which the induced current in (i) above
can be increased (2 marks)

(c) A transformer with 1200 turns in the primary circuit and 120 turns
in the secondary circuit has its primary circuit connected to a 400V a.c
source. It is found that when a heater is connected to the secondary
circuit, it produces heat at the rate of 600w. Assuming 100% efficiency,
determine the:

(i) Voltage in the secondary circuit (2 marks)


(ii) Current in the primary circuit (2 marks)
(iii) The current in the secondary circuit (1 mark)

19. 2008 Q18 P2


(a) Figure 12 shows two circuits close to each other

Coils

Figure 12

When the switch is closed, the galvanometer shows a reading and


then returns to zero. When the switch is then opened, the
galvanometer shows a reading in the opposite direction and then
returns to zero. Explain these observation
(3 marks)
(b) Explain how energy losses in a transformer are reduced by having:
(i) A soft- iron core (2 marks)
(ii) A laminated core (2 marks)

(c) An ideal transformer has 2000 turns in the primary circuit and 200
turns in the secondary circuit. When the primary circuit is connected
to a 400V a.c. source, the power delivered to a resistor in the secondary
circuit is found to be 800W. Determine the current in:
(i) The secondary circuit
(ii) The primary circuit

20. 2009 Q10 P2


Figure 5 shows magnet being moved towards a stationary solenoid.
It is observed that a current flows through the circuit in a direction Q TO P.

43
Explain:
(i) How the current is produced (2 marks)
(ii) Why the current flows from Q to P (1 mark)

21. 2010 Q6 P2
Explain why electric power is transmitted over long distances
at high voltages. (2 marks)

22. 2010 Q6 P2
Figure 6, shows a horizontal conductor in a magnetic field parallel to
the plane of the paper.

conductor

State the direction in which the wire may be moved so that the induced
current is in the direction shown by the arrow. (1 mark)

23. 2011 Q17 P2


(a) State what is meant by the term “electromagnetic induction” (1 mark)
(b) figure 9 shows asimple electric generator

coil
Motion
Magnet

Magnet N

S Q
R

Figure 9 P

(i) Name the parts labeled


P and Q. (2 marks)

(ii) Sketch on the axes provided, a graph to show how the magnitude of
the potential difference across R, changes with the time t. (1mark)

44
(iii) State two ways in which the potential difference produced by such
a generate can be increased. (2 marks)

(c) In a transformer, the ratio of primary turns to the secondary turns is1:10.A
current of 500mA flows through a200Ώ resistor in the secondary circuit.
Assuming that the transformer is 100% efficient, determine:

(i) The secondary voltage (1 mark)


(ii) Primary voltage

24. 2012 Q16 P2


(a) Figure 15, shows two coils A and B placed close to each other. A is
connected to a steady D.C. supply and a switch, B is connected to
a sensitive galvanometer.

Coil A Coil B Sensitive


galvanometer

Figure 15

i. The switch is now closed. State the observation made on the galvanometer
(2marks)
ii. Explain what would be observe dif the switch is then opened (2marks)

b) The primary coil of a transformer has 1000 turns and the secondary coil
has 200 turns .The primary coil is connected to a 240V a.c.. mains, supply

i. Explain how an e.m.f is induced in the secondary coil. (2marks)


ii. Determine the secondary voltage (2marks)
iii. Determine the efficiency of the transformer given that the current in
the primary coil is 0.20A and in the secondary coil it is 0.80 A. (3marks)

25. 2013 Q8 P2
Figure 4 shows a simple transformer connected to a 12V A.C. source
and an a.c

W
12v V

Figure 4
By counting the number of turns in each coil, determine the reading on
the volt meter. (3 marks)

45
26. 2014 Q15 P2
(a) One of the causes of energy loss in a transformer is heating in the
coils when current flows. State;

i) The reason why the current causes heating. (1 mark)


ii) How the heating can be minimized (1 mark)

(b) The input voltage of a transformer is 240 V and its output is 12 V. When
an 80 W bulb is connected across the secondary coil, the current in
the primary coil is 0.36A.Determine:

Np
(i)The ratio of the transformer, (where Np is the number of turns in
Ns
the primary coil and Ns is the number of turns in the secondary coil)
(3 marks)

(ii) The power input of the transformer (3 marks)

(iii)The power output of the transformer (1 mark)

(iv)The efficiency of the transformer (2 marks)

27. 2016 Q15 P2


(a) Figure 15, shows two coils A and B placed close to each other. A is
connected to a steady D.C. supply and a switch, B is connected to
a sensitive galvanometer.

Coil A Coil B Sensitive


galvanometer

Figure 15

iii. The switch is now closed. State the observation made on the galvanometer
(2marks)
iv.Explain what would be observe dif the switch is then opened (2marks)

b) The primary coil of a transformer has 1000 turns and the secondary coil
has 200 turns .The primary coil is connected to a 240V a.c.. mains, supply

i. Explain how an e.m.f is induced in the secondary coil. (2marks)


ii. Determine the secondary voltage (2marks)
46
iii. Determine the efficiency of the transformer given that the current in
the primary coil is 0.20A and in the secondary coil it is 0.80 A. (3marks)

MAINS ELECTRICITY
1. 1997 Q24 P1
How many 1000W electric irons could be safely connected to a 240V
main circuit fitted with 13A fuse? (3 marks)

2. 1999 Q34 P1
Figure 17 shows the electric wiring of an electric heater A, B, C are the
main wires.
A
B
C

Heater causing

Fig 17

Identify A, B, and C. (2 marks)

3. 1999 Q5(b) P2
An electric heater is made of a wire of resistance 100 and connected
to a 240V mains supply. Determine the:

i) Power rating of the heater (3 marks)


ii) Current flowing in the circuit. (2 marks)

iii) Time taken for the heater to raise the temperature of 200g of water
from 23oC to 95oC.(Specific heat capacity of water is 4200-1K-1) (4 marks)

iv) Cost of using the heater for two hours a day for 30 days.(The power
and lighting company charges Kshs 5.00 per kilowatt – hour). (4 marks)

4. 2001 Q32 P1
A 60W bulb is used continuously for 36 hours. Determine the energy
consumed. Give your answer in kilowatt hour (kWh).

5. 2002 Q18P1
An electric bulb with a filament of resistance 480 is connected to a
240V mains supply.
47
Determine the energy dissipated in 2 minutes (3 marks).

6. 2002 Q30 P1
The following table shows electrical appliances to be used in a house.
The electrical rating for each appliance is shown. The following fuses
are available, 5A, 15A, 30A and 45A.

Appliances Voltage (V) Power (W)


T.V 250 300
Iron box 250 750
Electrical kettle 250 2,000

Determine which one of the fuses is suitable for the house. (2 marks)

7. 2003 Q17 P1
An electric bulb is rated 75W, 240V. Determine the resistance of the bulb

8. 2004 Q20 P1
An electric heater is connected to the mains supply. A fault in the mains
reduces the supply potential slightly.
Explain the effect on the rate of heating of the heater.

9. 2005 Q19 P1
A hair drier is rated 2500W, 240V. Determine its resistance. (3 marks)

10. 2006 Q21 P2


(a) Figure 12 shows a section of a house wiring system

Socket
P

Cooker

Eart
h
Distribution box
Figure 12
(i) Name:
The circuit labeled P (1 mark)
The terminals labeled X and Y (2 marks)

(ii) I State the purpose of R in the circuit (1 mark)


II Give a reason why R is connected to Y but not to X (1 mark)

(iii) Why is the earthing necessary in such a circuit? (1 mark)

48
(b) Determine the cost of using an electric iron rated 1500W, for a total
of 30 hours given that the cost of electricity per kWh is Kshs 8. (2 mark)

11. 2010 Q6 P2
Explain why electric power is transmitted over long distances at
high voltages.

12. 2011 Q13 P2


State the reasons why electric power is transmitted over long distance
at a very high voltages. (1 mark)

13. 2012 Q12 P2


State why alternating current (a.c) is used for transmitting electricity
over long distances (1 mark)
14. 2012 Q15(c) P2
A house has one 100 W bulb, two 60 W bulbs and one 30W bulb.
Determine the cost of having all the bulbs switched on for 70 hours,
given that the cost of electricity is 40 cents per kilowatt hour. (3marks)

15. 2012 Q16 P2


(a) Figure 15, shows two coils A and B placed close to each other. A is
connected to a steady D.C. supply and a switch, B is connected to a
sensitive galvanometer.

Coil A Coil B Sesnsitive


galvanometer

Figure 15

i. The switch is now closed. State the observation made on the


galvanometer (2 marks)
ii. Explain what would be observe dif the switch is then opened (2 marks)

b) The primary coil of a transformer has 1000 turns and the secondary
coil has 200 turns. The primary coil is connected to a 240V a.c.. mains,
supply
i. Explain how an e.m.f is induced in the secondary coil. (2 marks)

ii. Determine the secondary voltage (2 marks)

iii. Determine the efficiency of the transformer given that the


current in the primary coil is 0.20A and in the secondary coil
it is 0.80 A. (3 marks)

16. 2013 Q9 P2
In domestic wiring systems lamps in the circuit are required to be in
parallel and not series. State two reasons for this requirement.

17. 2015 Q13 P2


When a transformer is connected to an ac source, the output voltage is
49
found to be 24 V. If the power input is 200 W, determine the output current.
(Assume the transformer is 100% efficient). (3 marks)

17. 2015 Q13 P2


(b) Explain how a fuse safeguards electrical appliances against excessive
currents. (2 marks)

(c) A hair dryer is rated 2.5 kW, 240V. :

(i) Determine whether a 10 A fuse may be suitable for the hair dryer. (5 marks)
(ii) Determine the cost of using the hair dryer for 3 hours if the cost of
electricity is Ksh 0.80 per kilowatt hour. (2 marks)

CATHODE RAYS AND CATHODE RAY TUBE


1. 1996 Q32 P1
The control grids in a cathode Ray Oscilloscope (CRO) is used to control
the brightness of the beam on the screen. How is this achieved? (2 marks)

2. 1998 Q19 P1
State two differences between the cathode ray tube (CRT) of a TV and
the cathode ray oscilloscope (CRO) (2 marks)

3. 2000 Q35 P1
State and explain the effect of increasing the E.H.T in an x-ray tube on
the X-rays produced.

4. 2001 Q31 P1
Fig. 22 shows an electric generator. The points P and Q are connected to
a cathode ray oscilloscope (CRO).

coil Rotation

Voltage
(v)

N S

Brush Brush
Q
Accumulator Time (t)

Sketch on the axes provided the graph of the voltage output as seen on
the CRO Given that when t = 0 the coil is at the position shown in the figure.
(2 marks)
5. 2001 Q4a P2
Fig 3 shows the main features of a cathode ray tube (CRT) of a cathode ray
oscilloscope (CRO)

50
Electron beam

Vaccum

Figure 3
i) Describe how the electrons are produced in the tube. (2 marks)
ii) State and explain the function of the grid. (3 marks)

iii) State what would be observed on the screen if an a.c voltage


is connected across the y-plates. (1 mark)
iii) State how the deflection system of a television differs from
that of a CRO. (1 mark)

v) Give the reason why it is possible to have a wider screen in


the television set than on the C.R.O. (1 mark)

6. 2002 Q29 P1
The circuit figure 14 represents a simple radio receiver.

Signal

CRO
R

fig. 14

On the axes provided, sketch the waveform observed on the CRO for
the signal shown. (1 mark)

51
7. 2003 Q34 P2
Figure 18 shows the pattern produced by an A.C voltage on a CRO screen.

Fig. 18

On the same figure, sketch the pattern produced by the same voltage
when the time base is switched off.

8. 2004 Q32 P1
Figure 17 shows the appearance of an alternating signal on a screen
of a cathode ray oscilloscope.
V (v)

t(s)

Figure 17
On the same diagram, sketch the appearance of the signal when the
frequency is doubled and the voltage halved.

9. 2006 Q12 P2
Figure 6 shows a tube for investing the properties of a beam of electrons.
Use the information in the figure to answer questions 9 and 10

Anode

52
Bright spot

Phosphorescent paint

Hot cathode Fine hole Beam of electrons

Figure 6

What property of the beam of electrons show that the electrons


are travelling at a very high speed? (1 mark)

10. 2006 Q13 P2


The beam of electrons is subjected to a strong magnetic field which is
perpendicular to the path and into the paper. Sketch on the same figure, the
new path of electrons. (1 mark)

11. 2007 Q18 P2


(a) Figure 14 shows the features of a cathode ray tube

Cathode Florescent
x-plates screen
y-plates Vaccum

Beam
Anode

(i) Name the parts labeled A and B (2 marks)


(ii) Explain how the electrons are produced in the tube (2 marks)
(iii) State two functions of the anodes (2 marks)
(iv) At what part of the cathode ray tube would the time
be connected?
(v) Why is a vacuum created in the tube? (1 mark)

(b) The graph in figure 15 was obtained on a cathode ray oscilloscope


(CRO) screen when the output of an a.c generator was connected to
the input of the CRO. The time- base calibration of the CRO was set
53
at 20 milliseconds per centimeter and the y- gain at 5 volts centimeter.

Figure 15

(i) Determine the pick voltage of the generator (2 marks)


(ii) Determine the frequency of the voltage (3 marks)

(c ) On the same grid, redraw the graph for the same voltage when the time
base calibration is set at 40 milliseconds per centimeter and y- gain
at 10 volts per centimeter. (Show at least one complete cycle) (2 marks)

12. 2008 Q12 P2


A narrow beam of electrons in a cathode ray oscilloscope (CRO) strike
the screen producing a spot. State what is observed on the screen if
a low frequency a.c source is connected across the y-input of the CRO
(1 mark)
13. 2010 Q18a P2
Figure 14a, is a diagram of a cathode ray tube. M and N are parallel vertical plates.

+ high voltage
Screen seen
Figure 14(a) from front

Figure 14(b)

(a) When switch S is open, a spot is seen at the centre of the screen as shown in
figure 14(b).
(i) State what happens to the spot when S is closed. (1 mark)

(ii) State what would happen to the spot if the potential difference
across MN is increased. (1 mark)

54
(iii) State what would be seen on the screen if the battery is replaced
with an alternating emf of:
(I) a low frequency of about 1 Hz; (1 mark)
(II) a high frequency of about 50Hz. (1 mark)

(b) Explain the process by which electrons are produced at F. (2 marks)

(c) State with a reason how the brightness of the spot can be increased. (2 marks)

(d) The accelerating voltage of the tube is 1000V and the electron current
in the beam is 1.5mA. Determine the energy conveyed to the screen
per second. (2 marks)

14. 2011 Q18 P2


(a) State two differences between cathode rays and electromagnetic radiations.
(b) Figure 10 shows the main features of a cathode ray oscilloscope (CRO).
M Electron beam
Heated fillament

Spot

Cathode

Florescent
screen

Figure 10

(i) Name the parts labeled M and N. (2 marks)


(ii) Explain how electrons are produced in the tube. (2 marks)

(iii) When using the CRO to display waveforms of voltages, state where the
following should be connected:
(I) the voltage to be displayed on the screen (2 marks)
(II) the time base voltage. (2 marks)

(iv) State why the tube is highly evacuated. (2 marks)

(c) Figure 11, shows the


waveform of a voltage displayed on the screen
of a CRO. The Y-gain calibration was 5V per cm.

55
(i) Determine the peak-to-peak voltage of the Y-input. (2 marks)

(ii) Sketch on the same figure the appearance of the waveform after the
the input signal is halved and it's frequency is doubled. (2 marks)

15. 2013 Q17 P2


(a) Figure 9 shows a cathode ray tube in which a beam of electrons is
cast on the screen.
Anodes
Grid
Cathode Screen

- 600V

- 450v

High voltage
supply
Figure 9

(i) State how the electrons are produced in the tube. (1mark)
(ii) State how the electron beam is detected (1mark)

(iii) State the reason for having a variable potential difference (p.d.) at the:
(I) Grid; (1mark)
(II) Anodes (1mark)
(b) Figure 10 shows the waveform of a signal applied at the y-plates of an
oscilloscope whose time-base is switched to the scale of 2 milliseconds
per centimeter.

56
Figure 10

Determine:
(i) The period of the signal; (2marks)
(ii) The frequency of the signal. (3marks)

16. 2014 Q13 P2


The current of electrons hitting the screen of a cathode ray oscilloscope
is 2.0 x 10- 4 A. determine the number of electrons that strike the screen
each second. (take charge of an electron as 1.6 x 10 -19 C) (3 marks)

17. 2015 Q11 P2


State one application of thermionic emission. (1 mark)
18. 2015 Q12 P2
Figure 4 shows a cathode ray entering into a region between
two charged plates.

Complete the diagram to show the path of the ray in the field.

X-RAYS
1. 1995 Q28 P1
Name the metal used to shields X – rays operators from the radiation.
Give a reason why it is used. (2 marks)

57
2. 1998 Q4 P1
State one industrial use of X – rays (2 marks)

3. 1999 Q23 P1
State the energy transformation when fast moving electrons are
suddenly stopped by a target in an X- ray tube. (1 mark)

4. 2000 Q35 P1
State and explain the effect of increasing the E.H.T in an x-ray tube
on the X-rays produced. (2 marks)

5. 2001 Q33 P1
State the factor that determines the hardness of the X – rays produced
in an X – ray tube. (1 mark)

6. 2002 Q32 P1
State the property of X-rays, which makes it possible to detect cracks
in bones. (1 mark)

7. 2003 Q25 P1
Give a reason why the target in an X-ray tube is made of tungsten or
molybdenum. (1 mark)

8. 2004 Q28 P1
State the difference between X-rays and Gamma rays in the way in
which they are produced.

9. 2004 Q33 P1
State the difference between hard X-ray and soft X-rays. (1 mark)

10. 2005 Q33 P1


The target of X-ray tube is made of metals of high melting point.
Give a reason for this (1 mark)

11. 2006 Q14 P2


State with a reason the effect on the X- rays produced in an X- ray tube,
when the p.d across the tube is increased (2 marks)

12. 2008 Q13 P2


The accelerating potential of a certain X-ray tube is increased. State
the change observed on the X-rays produced. (1mark)

13. 2008 Q19 P2


(a) X- rays are used for detecting cracks inside metal beams
(i) State the type of the X- rays used (1 mark)
(ii) Give a reason for your answer in (i) above (1 mark)

(b) Figure 13 shows the features of an X- ray tube

- High p.d +
Land
shielding Target

58
Cooling
firms

Oil

Filament Electron
beam
A
B
x-rays
Figure 13

(i) Name the parts labeled A and B (2 marks)

(ii) Explain how a change in the potential across PQ changes the intensity
of the X- rays produced in the tube. (2 marks)

(iii) During the operation of the tube, the target becomes very hot.
Explain how this heat is caused (2 marks)

(iv) What property of lead makes it suitable for use as shielding material?
(1 mark)
(c) In a certain X- ray tube, the electrons are accelerated by a Pd of
12000V. Assuming all the energy goes to produce X- rays, determine
the frequency of the X- rays produced. (Plank’s constant
h= 6.62 x 10-34 Js and charge on an electron, e = 1.6 x 10-19C). (4 marks)

14. 2009 Q11 P2


In an x-ray tube, it is observed that the intensity of x-ray increases
when potential difference across the filament is increased. Explain this
observation. (3 marks)

15. 2010 Q12 P2


An x-ray tube produces soft x-rays. State the adjustment that may
be made so that the tube produces hard x-rays. (1 mark)

16. 2012 Q18a, b P2


Figure 18 shows the parts of an x-ray tube.

59
a) Explain why;
i). A potential difference is applied to the filament (2 marks)

ii.)A high potential difference is applied between the cathode


and the anode (2 marks)

iii) Most of the tube is surrounded by lead. (1 mark)

b) State how the resulting x-rays are affected by increasing the potential
difference between the anode and the cathode (1 mark)

17. 2013 Q12 P2


Figure 5 shows a narrow beam of x-rays passing between two metal
plates in air. The plates are connected in series with a switch, a cell and
a milliameter.

Plate
x-rays
Plate

mA

Figure 5

It is observed that when the switch is closed a current flows in the


milliameter. Explain this observation. (2marks)

18. 2014 Q11 P2


60
The anode of an x-ray tube becomes hot when the tube is in use. State
the reason for this. (1 mark)

19. 2016 Q10 P2


Figure 5 shows a narrow beam of x-rays passing between two metal
plates in air. The plates are connected in series with a switch, a cell and
a milliameter.

Plate
x-rays
Plate

mA

Figure 5

It is observed that when the switch is closed a current flows in the


milliameter. Explain this observation. (2marks)

61
PHOTO ELECTRIC EFFECT
1. 1995 Q27 P1
An electron in an excited atom falls from energy levels E2 to energy
level E1. Write an equation relating the energy change to the frequency
f, of the radiation emitted. Explain why new symbols used. (2 marks)

2. 1995 Q29 P1
In an experiment on photo- electricity using metal X, the graph shown
in figure 9 was obtained. Use the graph to answer questions 2 and 3.
Stopping potential Vs(v)

Frequency f (Hz)

2. Determine the minimum frequency f0 below which no photoelectric


emission occurs (2 marks)

3. Sketch on the same axes, a graph for a metal, Y whose work function
is higher than metal X (1 mark)

3. 1996 Q7a,b P2
(a) Describe with the aid of a diagram an experiment set up for observing
photoelectric effect (4 marks)

(b) Table 2 shows the relationship between the wavelength,  of a


radiation falling on the surface and the energy, k of the emitted electrons

Stopping Potential, Vs(V) 0.7 0.9 1.1 1.4 1.6


Frequency, f (x1015Hz) 4.0 5.0 6.0 7.0 8.0

a. Plot a graph of energy stopping potential, Vs, against the frequency,


f, of the emitted electrons (5 marks)
b. Determine the work function WO of the surface used (6 marks)

62
Speed of light, c = 3.00 x 108 ms-1 planks constant h = 6.663 x 10-34JS]

4. 1997 Q5c P2
Electrons emitted from a metal when light of a certain frequency is shone
on the metal are found to have a maximum energy of 8.0 x 10-19J. If the
work function of the metal is 3.2 x 10-19J, determine the wavelength of
the light used. (5 marks)

5. 1998 Q4b P2
Fig 4 shows a photocell

Fig 4

(i) Label the cathode and anode. (1 mark)


(ii) How are electrons produced in the cell (1 mark)
(iii) Draw a simple circuit including the photocell to show the
direction of flow of current (2 marks)

(iv) Calculate the photon energy in ultraviolet radiation whose


frequency is 8.60 x 1014 HZ. (Plank’s constant h=6.63 x 10-34 Js)
(3 marks)
6. 1999 Q30 P1
It is observed that when ultraviolet light is shone onto a clean zinc plate
connected to the cap of negatively charged leaf electroscope, the leaf
collapse. Explain this observation. (3 marks)

7. 1999 Q7 P2
a) Figure 7 shows a photoelectric cell circuit:

Light source

Fig. 7

The intensity of the light can be varied.


i) Describe how the circuit may be used to show how the current
I varies with the potential difference V across the cell. (3 marks)

ii) Sketch on the same axis graphs of I versus V for three different
values of light intensity E1E2 and E3 such that E3>E2>E1 (3 marks)

63
b) Using a circuit similar to the one in figure 7. with the polarity of the
batteries reversed, the frequency, of the light was varied at constant
intensity. For each frequency, the potential difference was varied until
the current was equal to zero. The value of this voltage, Vco was noted.
The graph in figure 8 shows the relation between Vco and the frequency ,
of the incident light.Feom the graph, determine the:
Vco(v)

Frequency (f) x 1016Hz


Figure 8

i) Value of planks constant, h (charge an electron e = 1.6 x 10 -19Colubomb)


(5 marks)
ii) Work function,  , of the cathode surface of the cell . (Give your
answers to 2 decimal places.) (4 marks)

8. 2000 Q30 P1
In fig. 18 ultra – violet (u,v) light falls on a zinc plate placed on a
charged leaf electroscope. It is observed that the leaf collapses.

Zinc plate
u.v rays
Leaf electroscope

Leaf
Fig. 18

Explain how this observation may be used to determine the type


of charge on the electroscope (2 marks)

9. 2001 Q4 (b) P1
In an excited hydrogen atom. An electron moves from an energy level

64
of -1.36 x 10 -19J. Determine the wavelength of the radiation emitted.
(Planks constant h= 6.63 x 10-34 Js and speed of light c= 3.0 x 108 ms-1).
(5 marks)

10. 2002 Q34 P1


Light of a certain wavelength strikes the surface of a metal. State what
determines the maximum kinetic energy of the electron emitted. (1 mark)

11. 2002 Q7 P2
(a) Fig. 8 shows ultra violet light striking a polished zinc plate placed
on a negatively charged gold – leaf electroscope.

Zinc plate
u.v

Explain the following observations


i) The leaf of the electroscope falls. (2 marks)

ii) When the same experiment was repeated with a positively charged
electroscope the leaf did not fall. (2 marks)

B i) State two factors which determine the speed of photoelectrons


emitted by a metal surface (2 marks)

ii) In an experiment using a photocell, u, v. light of varying frequency but


constant intensity was made to strike a metal surface. The maximum
kinetic energy (K.E max)of photoelectrons for each frequency, was
measured. The graph shows how KE max varies with f.

Given that KE max = hf -  , determine the value of the constants h


and  form the graph. (6 marks)

c) Light of frequency 5.5 x 1014 Hz is made to strike a surface whose


work function is 2.5eV.Show that photoelectric effect will not take
place.(Use the value of h from (b) above. (3 marks)

12. 2003 Q35 P1


The minimum frequency of radiation necessary to cause photoelectric
effect on a certain metal surface in 9.06 x 10-14 Hz. Determine the work
function of the metal. (Planks Constant h=6.63 x 10-34Js) (3 marks)

13. 2004 Q34 P1


The work function of a certain material is 3.2 eV. Determine the
threshold frequency for the material. (1 electron Volt (eV)= 1.6 x10 -19)
and planks Constant h=6.62 x 10-34Js) (3 marks)

14. 2004 Q7 P2

65
a) i) What is photoelectric effect? (1 mark)

ii) You are provided with the following:

a photocell, a source uv light, a rheostat, a source of e.m.f, a


millimeter, a voltmeter and connecting wires.

Draw a circuit diagram to show how photoelectric effect may be


demonstrated in the laboratory. (1 mark)

b) In a photoelectric effect experiment, a certain surface was illuminated


with radiation of different frequencies and the stopping potential
determined for each frequency.

Table 2 shows the results obtained.

Frequency, f (x1014 HZ) 7.95 7.41 6.88 6.10 5.49


Stopping Potential, Vs (V) 1.35 1.15 0.93 0.62 0.36

(i) Plot the graph of the graph of stopping potential (y-axis) against
frequency. (5 marks)

(ii) Determine Planks Constant, h, and the work function, Ø ,of


the surface given that
eVs = hf- hfo
where e= 1.6 x 10-19 coulomb and hf = Ø
fo is the lowest frequency that can cause photoelectric effect.
(5 marks)
c) A surface whose work function  = 6.4 x 10-19 joules is illuminated
with light of frequency 3.0 x 1015HZ

Find the maximum Kinetic energy of the emitted photoelectrons


(Use the Value of H obtained in b(ii) (3 marks)

15. 2006 Q10 P2


Explain how an “excited’ hydrogen atom is able to emit radiations of
different wavelengths. (2 marks)

16. 2006 Q10 P2


What is meant by the term work function? (1 mark)

17. 2006 Q11 P2


If the frequency of the illuminating radiation is just equal to the threshold
frequency of the surface, explain why no photoelectric effect is observed
(2 marks)
18. 2007 Q11 P2
The graph in figure 8 shows the variation of photoelectric current with
applied voltage when a surface was illuminated with light of a certain
frequency. Use the information in the figure to answer questions 18
and 19.
Photoelectric
current I

V(v)
66
Figure 8

On the same axes, sketch the graph when light of higher intensity but same
frequency is used to illuminate the surface. (1 mark)

19. 2007 Q12P2


Explain your answer in 11 above (1 mark)

20. 2009 Q18 P2


(a) It is observed that when ultra-violet radiation is directed onto clean
zinc plate connected to the cap of a negatively charged leaf electroscope,
leaf falls.

(i) Explain this observation. (2 marks)


(ii)State why this observation does not occur if the electroscope
is positively charged.

(iii)Explain why the leaf of the electroscope does not fall when
infra-red radiation tested onto the zinc plate (1 mark)

(b) State the effect on the electrons emitted by the photoelectric when;
(i) The intensity of incident radiation I increase (1 mark)
(ii) The frequency of the incident radiation is increased

(c)The maximum wavelength of light required to cause photoelectric


emission on a metal surface is 8.0x10m..The metal surface is irradiated
with light of frequency 8.5x10 Hz.

Determine;
(i) the threshold frequency (2 marks)
(ii) the work function of the metal in electron volts (3 marks)
(iii)The maximum kinetic energy of the electrons. (2 marks)

Take: IeV=1.6 x 10 J
Speed of light =3.0 x 10 Js
Planks constant, h =6.663 x 10 Js

21. 2010 Q19 P2


(a) State the property of radiation that determines the number of
electrons emitted when a radiation falls on a metal surface.

67
(b) Figure 15 is a graph of the stopping potential Vs against frequency in an
experiment on photoelectric effect.

(i) What is meant by stopping potential? (1 mark)

(ii) Given that the stopping potential Vs is related to the frequency by the equation
h ωo
V s= f − Where e is the charge of an electron, (e = 1 .6 x 10-19C)
e e

Determine from the graph:


(I) plank's constant, h; (4 marks)
(II) the work function, ω o for the metal in electron volts (eV). (3 marks)

68
22. 2011 Q14 P2
State the meaning of the term “threshold frequency” as used in
photoelectric emission (1 mark)

23. 2012 Q18c P2


Light of frequency 7.5 x 104 Hz strikes a metal surface whose work
function is 4.0 x 10-19J. Determine the kinetic energy of the emitted
photoelectrons.
(take planks constant h=6.63 x 10-34 Js) (4 marks)

24. 2013 Q12 P2


A photon of ultraviolet light having energy E falls on a photoemissive
surface whose work function is T. Write an expression for the maximum
kinetic energy of the resulting photoelectron in terms of E and T. (1mark)

25. 2014 Q6 P2
It is observed that when the cap of an uncharged electroscope is
irradiated with light of high frequency, the leaf of the electroscope
rises. Explain this observation. (3 marks)

26. 2015 Q14 P2


(a) State two factors that affect photoelectric emission. (2 marks)

(b) Light of wavelength 4.3 x 10-7m is incident on two different metal


surfaces, nickel and potassium. (Take speed of light as 3.0 x 108 ms-1
and planks constant h as 6.63 x l0-34Js).

(i) Determine the energy of the incident radiation. (3 marks)

(ii) If the work function of nickel is 8.0 x 10-19J and that of potassium is
3.68 x 10-19J, state with a reason from which of the two metals the
given light will eject electrons. (2 marks)

(iii) Determine the velocity of the emitted electrons from the metal
surface in b(ii).
(Take the mass of an electron as 9.1 x 10-31 kg). (3 marks)

69
RADIOACTIVITY
1. 1995 Q32 P1
A radioactive carbon 14 decay to Nitrogen by beta emission as below

14 x 0
6
C
7
N+
y
e
Determine the values of x and y in the equation (2 marks)

2. 1995 Q32 P1
α- particles are more ionizing than - particles. Give one reason for this (1 mark)

3. 1997 Q34 P1
A lithium atom has 3 protons in its nucleus. Complete the diagram in
Figure 14 by marking X in the appropriate shells show the electron
distribution when the atom is not excited

4. 1997 Q35 P1
In a sample there are 5.12 x 10-20 atoms of krypton – 92 initially. If
the half of krypton; 92 is 3.0s determine the number of atoms that
will have decayed after 6s. (3 marks)

5. 1998 Q12 P1
The activity of a radioactive substance, initially at 400 counts per second
reduces to 50 counts per second in 72 minutes. Determine the half – life
of the substance.
(3 marks)
6. 1999 Q7 P1
In an experiment to study the atoms of gold, a beam of - particles
was directed onto a thin sheet of gold. The following observations
were made:

(i) Majority of the particles went straight through undeflected


(ii) A few particles deflected through varying angles up to 180. (3 marks)

70
7. 1999 Q35 P1
A radioactive nuclide of atomic number z emits a beta particle and
gamma rays. State the atomic number of the new nuclide. (1 mark)

8. 1999 Q6c P2
i) Define half – life of a radioactive material. (1 mark)
ii) Figure 6 shows a graph of the variation of the number of atoms
of a certain radioactive material with time.
Number of atoms

Time in minutes
Figure 6

Determine the half – life of the material (2 marks)

9. 2000 Q36 P1
The graph in Fig 20 shows the disintegration per second versus time
in seconds, s for a sample of radioactive material.

71
Disotegrations per second

Time (s)

Determine the half – life of the sample. (3 marks)

10. 2001 Q34 P1


The following reaction is part of a radioactive series:
210 210 c
X r Y α Z
83 84 b

Identify the radiation r and determine the values of b and c. (3 marks)

11. 2002 Q27 P1


Cobalt 60 is a radio isotope that has a half – life of 5.25 years. What
fraction of the original atoms in a sample will remain after 21 years? (3 marks)

12. 2002 Q31 P1


107
A nucleus is represents by X State the number of neutrons in the nucleus.
42
(1 mark)
13. 2002 Q2 P2
Fig. 3. Shows the path of radiation form a radioactive source after entering
a magnetic field. The magnetic field is directed into the paper and is
perpendicular to the plane of the paper as shown in the figure.

72
Source
Magnetic
field

Radiation

Figure 2

Identify the radiation and give a reason (1 mark)

b) Below is a nuclear reaction


232 A 228 B YC
k γ gamma
90 88 X

i) Identify radiation K (1 mark)


ii) Determine the values of X and Y. (2 marks)

(c) Fig 3 shows a device for producing metal foils of constant thickness.
Any change in thickness can be detected by the Geiger tube and recorded
by the Geiger. The pressure adjusted by the roller is then adjusted to
keep the thickness constant.

To Geiger
counter
Rollers

Direction of
movement Geiger tube
Metal foil

Thick metal
Radio active
sheet
source

Figure 3
(i) State the change in the metal foil that will lead to a decrease
in the Geiger counter reading (1 mark)
(ii) Give a reason for your answer in c(i) above (1 mark)
(iii) State the change in the roller pressure that should be made
as a result of this decrease in the Geiger counter reading. (1 mark)
(iv) Give a reason for your in (c) (iii) above (1 mark)
(v) Explain why a source emitting α (alpha) particles only
would not be suitable for this device. (2 marks)
73
(vi) Explain why a radioactive source of a half-life of 1600 years is
more suitable for use in the device than one of a half-life of
8 minutes. (2 marks)
14. 2003 Q18 P1
The following equation shows part of a radioactive decay process.

234 234
Th x Pa
90 91
Name the radiation x. (1 mark)

15. 2003 Q4(b) P2


Figure 4 shows a Geiger muller (GM) tube

Cathode (metal cylinder

High potential
Mica window
Anode
Figure 4 Argon gas at low pressure +
halogen vapour

(i) Give the reason why the mica window is made thin. (1 mark)
(ii) Explain how the radiation entering the tube through the
window is detected by the tube. (4 mark)
(iii) What is the purpose of the halogen vapour (1 mark)

16. 2004 Q3 P2
Figure 4 shows the cross-section of a diffusion cloud chamber used
to detect radiation from radioactive sources.

Transparent lid

Felt soaked in
Light source alcohol

Source
Base of chamber
Solid CO2

Foam

Figure 4

a) i) State one function of each of the following:


Alcohol
solid CO2 (2 marks)
74
ii) When radiation from the source enters the chamber, some white
traces are observed. Explain how these traces are formed and state
how the radiation is identified. (4 marks)

iii) A leaf electroscope can also be used as a detector of radiation.


State two advantages of the diffusion cloud chamber over the leaf
electroscope as a detector. (2 marks)

b) i) Two samples of the same radioactive material have initial masses


M and 2M respectively. On the axes provided, sketch the graph
of activity versus time for each sample. Label the graph for each sample.
(2 marks)

Activity (A)

0 Time (s)
ii) A radioactive sample of half-life 130 days initially has 1.0 x 10 20
radioactive atoms. Determine the number of radioactive atoms
that have decayed after 390 days. (3 marks)

17. 2005 Q36 P1


The following represents a nuclear reaction involving the nuclide
polonium Po

218 214 214 214


84
Po m 82
Pb n 83
Bi X 84
X

Identify m, n and X (3 marks)

18. 2005 Q7 P2
(a) What is meant by radioactivity (1 mark)
(b) With an aid of a labeled diagram explain the working of Geiger
Muller tube as a detector of radiation (5 marks)

(c) In an experiment to determine the half life of a certain radioactivity


substance, the activity in disintegrations per minute was measured
for some time. Table 1 shows the results obtained

Time in Minutes 0 10 20 30 40 50 60 70 80
Activity in disintegrations 152 115 87 66 50 38 20 12 6
Per minute

On the grid plot a suitable graph and sue it to determine the half life
t ½ of the substance (7 marks)
75
(d) At time t = 40 minutes, the activity of a sample of a certain
radioactive isotope with a half life 12 minutes if found to be
480 disintegration per minute.
Determine the time which activity was 3840 disintegrations per minute
(2 marks)
19. 2006 Q15 P2
A nuclear reaction is represented by the following equation

a 234
X Y + alpha particle
92 b

Determine the values of a and b (2 marks)

20. 2007 Q13 P2


The following is part of radioactive decay series

234 Bi a X 230 Y
β
84 84 b

Determine the values of a and b (2 marks)

21. 2008 Q14 P2


A radioactive isotope of copper decays to form an isotope of Zinc
as shown below

69 69
Cu Zn + Radiation
29 30

Name the radiation emitted and give a reason for your answer (2 marks)

22. 2009 Q17 P2


(a) figure 11 shows the path of radiation from a radioactive source. The
field is perpendicular to the paper and directed out of paper.

Magnetic
field

Figure 11
Identify the radiation (1 mark)

(b)Radiation from a radioactive source enters a G.M tube


(i) State the effect of the radiation on the gas inside the tube. (1 mark)
(ii) Explain how the large discharge current is created. (2 marks)

(c) The following is a nuclear equation for fission process resulting from the
reaction of a neutron with a uranium nucleus
76
1 235 141 y 1
n+ U A + Q+3 n
0 92 56 x 0

(i)determine the values of x and y. (2 marks)


(ii)State the source of the energy released (1 mark)
(iii)Explain how this reaction is made continuous in a nuclear reactor.
(2 marks)
23. 2010 Q9 P2
The initial mass of a radioactive substance is 20g. The substance has a
half-life of 5 years. Determine the mass remaining after 20 years.

24. 2011 Q19a, b P2


(a) When a radiation was released into a diffusion cloud chamber,
short thick tracks were observed. State with a reason the type of
radiation that was detected. (2 marks)

(b) The half-life of an element X is 3.83 days. A sample of this element


is found to have an active rate of 1.6 × 103 disintegrations per
second at a particular time.

Determine its activity rate after 19.15 days. (2 marks)

25. 2012 Q6 P2
Figure 6, shows a narrow beam of radiation from a radioactive source,
incident to a postcard. The emergent radiation passes through a magnetic
field which is perpendicular to the plane of the paper, and into the paper.

Source

Postcard Magnetic field


Figure 6

A detector moved along line AC detects radiations only at points B and


C. state the two types of radiations detected (1 mark)

26. 2013 Q7 P2
The equation below represents a nuclear reaction in which two deuterium

77
nuclei fuse to form Helium and X

2 2 3 a
H+ H → He+ X
1 1 2 b

(a) Determine the values of a and b (1mark)


(b) Identify X (1mark)

27. 2014 Q9 P2
Radium undergoes radioactive decay by emitting an alpha particle to
form a daughter nuclide Q as in the reaction:

226
88 Ra Alpha particle + XY Q

Determine the values of:


(a) X (1 mark)
(b) Y (1 mark)

28. 2016 Q8 P2

The following is part of radioactive decay series

234 Bi a X 230 Y
β
84 84 b

Determine the values of a and b (2 marks)

29. 2016 Q17a, b P2

(a) When a radiation was released into a diffusion cloud chamber,


short thick tracks were observed. State with a reason the type of
radiation that was detected. (2 marks)

(b) The half-life of an element X is 3.83 days. A sample of this element


is found to have an active rate of 1.6 × 103 disintegrations per
second at a particular time.

Determine its activity rate after 19.15 days. (2 marks)

78
ELECTRONICS
1. 1995 Q7 P1
Describe how a p- type semi conductor is formed (3 marks)

2. 1997 Q33 P1
Using the components symbols shown in figure 14, sketch a series
circuit diagram for a forward biased diode. (1 mark)

3. 1997 Q6 P2
(i) Distinguish between semiconductor and conductors (3 marks)
(ii) Give one example of a semiconductor and one for a conductor (3 marks)

4. 1998 Q21 P1
79
In the circuit in fig 5 when the switch S is closed, the voltmeter shows
a reading.

Cell

When the cell terminals are reversed and the switch is closed, the
voltmeter reading is zero. Explain these observations.

5. 2000 Q37 P1
What is meant by donor impurity in semi conductors (1 mark)

6. 2000 Q7(a) P2
Explain how a p- type semiconductor is made from a pure a semiconductor
(3 marks)
7. 2001 Q5a,b P2
a) You are provided with 12V a.c source, four diodes and resistor.
i) Draw a circuit diagram for a full wave rectifier and show the
points at which the output is taken. (5 marks)

ii) Sketch the graph of the output when a capacitor is put in parallel
with the resistor in the circuit in (i) above. (1 mark)
V

t
8. 2003 Q19 P1
Pure silicon can be changed into p –type semiconductor by adding
an impurity. Explain how this is achieved. (3 marks)

9. 2005 Q35 P1
Draw appropriate symbols the circuit diagram of a junction diode in
reverse bias.
(1mark)
10. 2005 Q4 P2
(a) Explain how doping producers an n-type semi-conductor for a
pure semi- conductor material. (3 marks)

b) Fig 5. Shows the circuit of a rectifier using four diodes D1, D2, D3
and d4.

80
Figure 5

(i) Explain how a rectified output is produced from the set – up


when an a.c input is connected across AB (4 marks)
(ii) On the axis provided sketch the graph of output voltage against
time for rectifier (1 mark)
(iii) A capacitor is now connected across XY. Explain the effect of
the capacity on the output. (2 marks)

11. 2006 Q16 P2


In the axes provided sketch the current – voltage characteristics
for reverse – biased p- n junction (1 mark)

- +
v

12. 2006 Q7 P2
(a) Explain how a p- type semiconductor is made from a pure a Semiconductor

(b) The curves in fig 10. Show the output characteristics of a n – p-n transistor in
common emitter mode. The p.d of the battery, V0 is 9.0V and the load
resistors RL is 1.8 k W

Fig. 10
I8=70μA

81
I8=50μA

L8(mA)
I8=30μA

I8=10μA

V(v)
Figure 10
i. Draw the circuit diagram for the experiment set- up that may be used
to obtain the curves in the figure.

ii. Given that ohm’s law for the circuit is VCE = Vcc – IcRL, draw on the same
axes, the load line for the circuit ( hint: load - line passes through.
(VCE = 0 and Ic = 0)
Drawing load line on graph (see graph)

When IB = 30μA, An alternating signal is fed into the base so that the base
current changes by ± 20μA. Use the graph to determine the corresponding
change in collector current Ic and hence determine the current gain b.

13. 2007 Q14 P2


You are provided with a diode, a resistor R, an a.c source of low voltage
and connecting wires.B
In the space provided, sketch the circuit diagram for a half – wave
rectifier and indicate the terminals where the output voltage V0 may be
connected. (2marks)

14. 2009 Q13 P2


Figure 6(a) and figure 6(b) shows a p-n junction connected to a battery. It is
observed that the current in figure 6(a) is greater than the current in figure 6(b).

p n
P n

Figure 6(a) figure 6(b)

State the reason for this observation. (1 mark)

15. 2010 Q14 P2


Figure 7, shows a block diagram of a p-n junction diode.

82
P-type n - type
On the same diagram, show how a battery may be connected so
that the diode is reverse biased. (1 mark)

16. 2011 Q19c,d P2


(a) State what is meant by an extrinsic semiconductor? (1mark)
(b) Figure 12,shows a depletion layer in an unbiased p-junction.

n-type p-type

-- - - - + - + + + +

Figure 12
- - - - + - + + + +

- - - - + - + + + +
Holes

electrons Depletion layer

Show how a battery can be used to make the depletion layer narrower. (1mark)

(c) Figure 13 shows an incomplete circuit of a full wave rectifier.

Figure 13

(i) Draw in the figure two more diodes to complete the circuit. (2mark)
(ii) Show on the figure the points across which the output of the
rectifier should be obtained. (1mark)

17. 2012 Q13 P2


Figure 11, shows an alternating current (a.c) connected across a
diode D and a resistor R.

83
To C.R.O
(output)

Figure 11

On the axes provided sketch the output as observed in the C.R.O connected
across R. (1mark)
V

18. 2013 Q13 P2


When a germanium crystal is doped with arsenic, it becomes an N-type
semiconductor. Explain how this change occurs.
(Number of electrons in the outermost shell for germamium = 4, Arsenic = 5).
(2marks)
19. 2014 Q8 P2
Figure 5 shows a graph of current against voltage for a semiconductor diode.
Current I(Ma)

Voltage v(v)
Figure 5

In the pace provided, draw a circuit diagram the may be obtain values
needed to draw the graph in figure 5. (3 marks)

20. 2016 Q12 P2


When a germanium crystal is doped with arsenic, it becomes an N-type
semiconductor. Explain how this change occurs.

84
(Number of electrons in the outermost shell for germamium = 4, Arsenic = 5).
(2marks)

21. 2016 Q17c,d P2


(c) State what is meant by an extrinsic semiconductor? (1mark)
(d) Figure 12,shows a depletion layer in an unbiased p-junction.

n-type p-type

-- - - - + - + + + +

Figure 12
- - - - + - + + + +

- - - - + - + + + +
Holes

electrons Depletion layer

Show how a battery can be used to make the depletion layer narrower. (1mark)

(c) Figure 13 shows an incomplete circuit of a full wave rectifier.

Figure 13

(iii) Draw in the figure two more diodes to complete the circuit. (2mark)
(iv) Show on the figure the points across which the output of the
rectifier should be obtained. (1mark)

MIXED CONCEPTS
1. 1998 Q27 P1
85
During total eclipse of the sun, both light and heat are observed to
disappear simultaneously. Explain the observation. (2marks)

2. 2000 Q25 P1
Fig. 3 shows a device for closing a steam outlet.

45cm
15cm
Pivot
Light bar
Piston W
Steam from boiler

Boiler
Fig. 3

The area of the position is 4.0 x 10-4 m2 and the pressure of the steam in the
boiler is 2.0 x 105 Nm3. Determine the weight W that will just hold the bar
in the horizontal position shown. (3marks)
3. 2005 Q3 P1
The light uniform bar in Fig 3 is equilibrium. The two beakers A and B
contain water at the same temperature. The two blocks are made of the
same material.

Water
Water

B
A
Figure 3

If the temperature of the water in beaker A is now raised, explain why the
beam tips to side A. Assume the solid does not expand. (2marks)

4. 2007 Q14 P1
Figure 8 shows a uniform light bar resting horizontally on corks floating

Light bar 86
on water in two beakers A and B.

cork B
A
water

Explain why the bar tilts towards side A when equal amount of heat is
supplied to each beaker (2 marks)

5. 2016 Q6 P1
Figure 8 shows a uniform light bar resting horizontally on corks floating
on water in two beakers A and B.
Light bar

cork
B
A
water

Explain why the bar tilts towards side A when equal amount of heat is
supplied to each beaker (2 marks)

THIN LENSES
87
MARKING SCHEME

1. 1995 Q3b P2

l=l+l l = 10 = 1
f u v u 60 6
l = l + l u = 6cm
10 u v
l = l+ l Objects is 6 cm from the lens
U 10 15

2. 1995 Q4 P2
(a) Lens symbol object between F1 & F2 appropriate rays position of image
Image correctly drawn

b) The diagram in figure 3 shows a certain eye defect

Near point

(b) (i) Name of defect is long sightedness


(Refer to the diagram in the figure 3 above)

3. 1996 Q38 P1

4. 1998 Q6 P1

5. 1998 Q7 P2
88
a) 2 complete rays, 2 with arrow at one end image (inverted real) (continuous tie)
locating F size 2.4 +0cm
b)
U (cm) 20 25 30 40 50 70
V(cm) 20 16.7 15 13.3 12.5 11.6
1 0.50 0.040 0.033 0.025 0.020 0.014
V(cm-1)
1 0.50 0.060 0.067 0.075 0.080 0.086
-1
V(cm )

ii) 1/f = 1/u +1/v Intercept 1/f


0.1 = 1/f ... f = 10cm

6. 1999 Q32 P1
Object should be between F and lens.

7. 1999 Q32 P1

89
8. 2000Q31 P1

9. 2001 Q30 P1

10. 2002 Q28 P1

11. 2003 Q16 P1

Rays marked independently: Correctly if in the right direction with arrows. Object
distance is 9.1cm + 0.2 (8.9 – 9.3). No arrow on the virtual. Any through optical
centre.
Other rays to principal axis and dotted through F.

12. 2003 Q7 P2
a) i) The candle is placed at a distance u from lens and screen position adjusted
until sharp image is obtained; the distance v between lens and screen is
measure; Process is repeated for other values of V;
For each set of u, v, f is found 1/f = 1/u + 1/v; average f determined;

(ii) Image is virtual and so not formed on screen

90
b) m = v= 2
v
/15 + 1/30;
=1/f = 1/15 + 1/30
F= 10cm

13. 2004 Q31 P1

14. 2005 Q30 P1


Image changes from real to virtual
Image changes from inverted to upright
Image changes from behind lens to the same side as object.
15. 2005 Q1 P2
With distance between lens and object being greater than facal length f;
(b) Adjust the lens distance until a sharp image of object is formed
besides object
(c) Distance between the lens and the object is measured and
repeated several times
(d) The average of the distance is the focal length of the lens

16. 2006 Q18 P2


(a) (i)

(ii) Image at 10cm from mirror (using scale)


(iii) Magnification
Size of image = 4.0 cm = 2
Size of object 2.0 cm
OR
Image distance = 2.0 cm = 2
Object distance 1.0 cm

91
(b) ( i) I Image distance
I=I+I
f v u
I=1 -I = 3
v 5 20 20

v = 20 = 6.67 cm
3
II Magnification
=v = 6.67 = 0.33;
u 20
(ii) Image characteristics: real, inverted, diminished, less bright

17. 2007 Q4 P2

18. 2007 Q10 P2


The process of the eye lens being adjusted to focus objects at various distances

19. 2007 Q16 P2

Diverging effects should be seen


(b) (i) A diaphragm
B Film

(ii) The distance between the lens and the film / object is adjusted; so that the
image is formed on the film
Adjust the shutter space/ adjust the aperture

92
(iii) Shutter – opens for some given time to allow rays from the object to fall on
the film creating the image impression/ exposure time is varied

A (diaphragm) controls intensity of light entering the camera


B (film) – coated with light sensitive components which react with light
to create the impression register/ recorded or where image is formed.
(c) (i) magnification = v/u = 3
Since v + u = 80
U = 80 – v
v =3
80 – v
V= 240 – 3v
V= 60cm (3 marks)
(ii) From above u = 20cm
l
/f = l/v + l/u = l/60 + l/20 (2 marks)
F = 15cm (15 marks)
20. 2008 Q5 P2
Needs not be dotted

21. 2008 Q11 P2


Short sightedness/ myopia
Extended eyeball/ lens has short focal length/ eye ball too long any two
22. 2009 Q9 P2
(L-q) cm

23. 2010 Q9 P2
(i) The following should be clearly shown: 2 rays, dotted extensions and the
image

1 1 1 1 1 1 400
(b) + = + = v= : U = 80 + 60 + 140
U V f 50 80 400 3
93
1 1 1 700 400 700 5
+ = V= ∴ length of image = - = 55 mm
V 50 700 3 3 3 9

24. 2011 Q16 P2


(a) Point where rays close and parallel to principal axis converge - convex lens Seem
(appeal) to diverge from - concave lens after striking the lens( I mark)

(b) (i) Diaphgram


(ii) Candle placed at certain distance from lens distance between the screen
and the lens adjusted until a sharp image is focused on the screen.
(iii) Distance of candle and lens (u) is measured
Distance of screen from lens (v) is also measured
(vi) values of V and U are substituted in the formula f = uv
u+v
(1/f = 1/u + 1/v ) or
• Use the graph of 1/v against 1/u and get reciprocal of intercept or measure L1
measure D(2 position of lens
• Use F=L2-D2
.4L
• Or graph of UV against u + v, gradient = f
(c) 1/v = 1/f – 1/u or m = v/u m = v/f
1
/v = 1/-20 – 1/30 = -12/30
V= -12cm = 0.4 (ignore –ve sign)

25. 2012 Q17(a) P2


(i) The image size decreases.
(ii)When m = I, image size = object size, hence u = 40cm.
(iii) u = 25cm, m = 3.5 and since m = v/u v = 25 x 3.5 = 87.5cm

26. 2012 Q9 P2

- each array√√
- correct virtual diminished image√

27. 2013 Q14 P2


(a) 10cm
(b) (i) Lens A, placed closer to the object, magnifies the object to an image that
serves as the object to lens B
(ii) Lens B is used for viewing. It magnifies the image produced by lens A to a
bigger virtual image.

(c) (i) The object is placed between F0 and 2F so that a real, inverted and
magnified image if produced,
94
(ii) The lens is adjusted so that the image falls between the principal focus aw
itself.

28. 2014 Q14 P2


Figure 1 shows two parallel rays from a distant object passing through a convex lens:

c) Indicate on the diagram, the position of the principal focus of the lens (1 mark)
d) Determine the focal length of the lens (1 mark)
10+0.5cm

29. 2015Q18b P2
1 1 1 1 1 1
(i) (I) u = 50mm, v = -100: = + = + = → f =100 mm
f u v 50 −100 100
v 100
(II) Magnification = (m) = = =2
u 50

(ii)

95
UNIFORM CIRCULAR MOTION
MARKING SCHEME

1. 1995 Q4 P2
(c) (i) For water not to pour weight of the water must be less centrifugal force
OR for water to pour out MV2 > mg
r
(ii) Frictional force F = Centripetal force
MV2 = 1200 x (25)2
R 150
3
= 5.0 x 10 N
2. 1996 Q6 P2
(a) Keep angular velocity Wl constant;
Centripetal force provided by mg;
Fix the mass m and measure of m;
Repeat for different values of m;

(b) (i) graph ( see on the next page


Axes labeled
Scale
Pts plot
Straight line

(ii) Gradient of the graph


= 0.625 – 0.1 = 1.167 N
0.525 – 0.075

Force F on the body = mbW2r


Where mb = mass of the body
Mbw2 r = Gradient of the graph = 1.167
W2 = 1.167 = 11.67
0.1
W =  11.67
= 3.42 rad s1
96
3. 1998 Q18 P1
Air resistance / Frictional force

4. 1998 Q29 P1
Since F=MV2/V the sharper the corner (as B) the small the value of R hence the
greater the F. (M& V constant).

5. 1999 Q31 P1
Tension = centripetal force.
T = Mv2/r but v = wr 2 = 0.1 x w2 x 0.33
2
T = Mw r t = 0.2 x 10 = 2N 2N = Mw2r 2 = 0.1 x w2 x 0.03
-w2 = 2/0.003w 2000/3 w = 666.7 = 25.82 rads/s

6. 2000 Q5 P2
(a) When whirled in air centripetal force is provided by bottom of container because
of the holes, there is no centripetal force on water on the water, so it escapes through
holes leaving clothes dry.

(b) (i) I Centripetal force equals force of friction


F= Mw2r = 0.4
W2 = 0.4 or F = Mw2r
0.1 x 0.08 0.4 0.1 w2 x 0.08
W= 7.07rad/s W = 7.07 rad/s

II F= Mw2r = 0.1 x 7.072 x 0.12


= 0.60N
Force required = 0.60 – 0.40
0.20N
(ii) The block will slide this is because although the frictional force is greater
centripetal force would be needed to hold it in place.

7. 2001 Q29 P1

Path

8. 2002 Q26 P1
a = w2r = (2Лf) 2r = 4 x π 2 x 82 x 0.2 = 505m/s2

9. 2003 Q2(b) P2
(i) potential energy lost = kinetic energy gained
1
mgh= mv 2
2
1
2 ×10 ×0.9= × v2 v=√ 18=4.24 m/s
2
iii) Force is centripetal = mv2
r
97
= 20 x 4.24
4
= 89.9V Total 14 marks
10. 2004 Q29 P1
Fr
F = mr2 V=√
m
r
81 ×0.2
V=√
0.5
= 9m/s
11. 2005 Q29 P1
(i) F = MV2/r
4800 = 800 x V2
20
V = 10.95m/s (allow 10.09 of a slide is used)

Alternatives.
(ii) Vmax = √Mrg but Fr = Mμg
M = Fr = 4800
Mg 800 x10 = 0.6
(iii) F = Ma
4800 – 800 x a, a = 6m/s2
A = v2/r
OR
6 = V2/20
V = 10.95m/s

(iv) F = MR, M = F/R = 4800


800 = 0.6
Tan ө = 0.6
V2 = rg tan ө
OR
2
V = 20 x 10 x 0.6
V = 10.95
12. 2005 Q6 P2
a) Angular velocity is the ratio of angle covered (angular displacement) to the
time interval
or ω= θ2 – θ1
t2 – t1
(b) ω = 300 – 170 = 10 radis-1
13
10t = 170
T = 17 sec
(c) (i)

98
(ii) T = mrω2r – C slope = mr = 1.5 – 0.25 = 0.061
28.5 – 8.0
M= 0.061 = 0.203 Kg (0.2 kg)
30 x 10-2
iii) Extent graph (calculate) C= 0.2
It represents frictions between table and body

13. 2006 Q10 P1


At F, radius of curve is smallest and so greatest centripetal force is required to
keep luggage on carrier; ( F =mv2)
R

14. 2007 Q18 P1

18.(a) Rate of change of velocity towards the centre 2 marks


Acceleration directed towards the centre of the motion
Acceleration towards the centre of orbit/ nature of surface
(b) Roughness / smoothness of surface. Radius of path/ angular 2 marks
(i) velocity/ speed
(Any two)
(ii) II) A> (l)B (l)C ( correct order) 1 mark
(c) F = m(l)2 r F = MV2 V=rw 4 marks
For thread to cut r w = 3.049
2
F= 5.6 N 5.6 = 0.2 x v 0.15
(l) = 13.7 radius V2 = 4.2 = 13.66
13.66 v = 2.0494

15. 2009 Q14 P1


V= r x 21 OR T = 1/33 = 0.030303
= 0.08 x 21 V 33 m/s T = 2V/ω =
=16.6 m/s ω= 2v/0.0303 = 207.525
V=rω
0.08 x 207.529
99
= 16.5876 m/s
16. 2009 Q19 P1
a) rate of change of angular displacement with time
Acc: without (rate)

b) i) mass, friction, radius. (any two)


ii) Oil will reduce friction since frictions provide centripetal force; the
frequency for sliding off is lowered.

V2 = U2 + 2as
= 0+ 2(0.28)h
V= √0.56 x 1.26
= rω
0.84 = 0.14 x ω = 0.84 = 6 rad/s
0.14
17. 2010 Q12 P1
F = mω2r = mg Or F = mv2/v but V = ωr
0.2 x 1 x ω2 = 0.5 x 10 ω2 = f/mr = 0.5 x 10/0.2 x 1
ω2 = 5/0.2 ω = 5 rad/s
ω = 2.5 = 5 rad/s

18. 2011 Q16(a) P1

(a) (i)
X String

Stone

(ii) 2m/s
(iii) the body obeys newton’s first law of motion. Due to its inertia no external force
acts on it

19. 2012 Q16c P1


(i) – weight of object; / gravitational force
- tensional force
(ii) A Object (1 mark)
String
r
Motion

20. 2013 Q16(a) & (b) P2


(a) • increasing the speed of rotation of the mass m
• reducing the radius of the circular path
(b) (i) Tension in the spring supporting the object
(ii)

100
(iii) There's change in the direction of instantaneous velocity at various
points along the circular path.
(iv) reading of spring balance = centripetal force
mv2 0.5 ×8 ×8
= = =16 N
r 2

21. 2014 Q18 P1


(a) Frictional force

(b)
 Increases the centripetal force acting on the bus
 Provide more centripetal force
 Prevent skidding force, overturning /rolling
 Enable higher speed / critical yield

(c)
(i)
The weight /force of gravity
The tension on the string

(ii) Determine the


(I) Angular velocity of the stone
3mks
V
W=2πf F=2 rev/s W=
R
2x2xπ 2 πr x 2 x 2
12.56 rad s-1 12.57 rad s-1

(II) 3mks
T;mv2-mg T+mg =mv 2
= 12.636-2
r r
Mw2r-mg T= mv2 – mg 10.6363
r
r2
0.2× ( 4 π 2 )
=10.63N = - mg
r

22. 2015 Q17d


(i)

101
(ii) The high speed of the air above the ball creates a region of low pressure above
it hence the higher atmospheric pressure below it pushes it upwards

23. 2015 Q18a P1

(i) - Centripetal force - weight


(ii) – centripetal forces increases
- weight of the body remains contant
(iii) – vehicle speed governor,
- centrifuges
- satelites

21. 2014 Q18 P1


(a) Frictional force

(b)
 Increases the centripetal force acting on the bus
 Provide more centripetal force
 Prevent skidding force, overturning /rolling
 Enable higher speed / critical yield

(c)
(i)
The weight /force of gravity
The tension on the string

(ii) Determine the


(I) Angular velocity of the stone
3mks
V
W=2πf F=2 rev/s W=
R
2x2xπ 2 πr x 2 x 2
12.56 rad s-1 12.57 rad s-1

(II) 3mks
2 2
T;mv -mg T+mg =mv = 12.636-2
r r
Mw2r-mg T= mv2 – mg 10.6363
r

102
r2
0.2× ( 4 π 2 )
=10.63N = - mg
r

FLOATING AND SINKING


MARKING SCHEME

2. 1996 Q29 P1
Solid copper is denser than water hence the solid sphere sinks; weight is greater
than up thrust. Hollow sphere experiences an up thrust equal to its weight so it
will float/ density of hollow sphere is less than that of water (2 marks)

3. 1998 Q2b P2
i) Volume of block = 4 x 4x16 = 256 cm3
Mass of block = 154 gm

D= m=154=0.6g/cm3 deny ½ mark if not to d.p


V 256

ii) Volume of liquid ¾ of 256 = 192 cm3


Density of liquid = 154 = 0.8g/cm3
192
4. 1999 Q29 P1
Up thrust = PV x 10 = 10 PV

103
5. 1999 Q6 P2
a) Benzene sinks in liquid benzene.
Water increases in volume on solidifying while benzene reduces in volume; ice
is less dense that liquid water. Solid benzene is denser that liquid benzene.

b i) Weigh the metal block in air and in water


Fill the overflow can in water and place on a bench / diagram
Collect the overflow in the beaker and weigh
Compare difference in weight of metal block and weight of overflow
Repeat
Up thrust = tension + weight
= (0.5 + 2.0) = 2.5N alternative
Weight of H2O) = 2.5N Up thrust = 2.5N
Mw = 1000 R.D = Wt. in air = 2.0 = 0.8
Vw Upthrust 2.5
Vw = 0.25 volume of wood €wood
1000 €wood
Density of wood = 0.2 €wood
0.25/100
0.2 x 1000
25
800kg/m3
c i) Time taken for half of the radio acute material to disintegrate.
i) Correct readings for 60 and 30 time 25 + 2 minutes

6. 2000 Q4b P2
(b) Volume of A displaced = 6.0 x 12 cm3 or P = G * g
Mass = 12 x 106 x 800 F=PxA
= 0.0096 kg ans = 0.09N
Weight = mg = 0.096N
(ii) Volume of B displaced = 6.0 x 3 = 18 cm3
Weight = 18 x 106 x 1000 x 10 = 0.18N

(iii) Weight of block = weight of third displaced


0.096 + 0.18 = 0.276
Mass = 0.027 kg
Volume = 0.0276 kg
42 x 10-6m3
=657 kgm-3 can also be in g/cm3

7. 2001 Q2 P1
Weight on side A has bigger volume when water is added.

8. 2001 Q22 P1

104
9. 2001 Q28 P1
Tension in A = 1.05N – 1.0N = 0.05N
Tension in B = tension due to A + Tension due to B
0.05 _+ 0.05 = 0.10N

10. 2002 Q25 P1


Law of floatation – a floating body displaces its own weight
Weight of block = weight of mercury displaced
0.250 x g = 13.6g
0.25 = v
3
13.6 x 10
V = 1.838 x 10-5 m3 = 18.4cm3
1.839 x 10-5m3

11. 2003 Q20 P1


Piece of metal does not displace own weight but the two together displace their
own weight/ weight of water displaced is less than the weight of metal while
weight of water displaced equals the weight of the tow/up thrust equal to combined
weight.

12. 2004 Q2 P1
Air in the balloon expands/volume of balloon increases; displaces more air
raising the up thrust of air;

13. 2004 Q1 P2
a) - Put in water and mark
-Put in liquid and Mark
- Space between the 2 marks which represent the reciprocals of densities is
divided into equal parts.

b)

105
i) Up thrust=0.49N
ii) Up thrust=weight of liquid displaced (Archimedes Principle) = 0.4N
Mass of Liquid =0.049kg=49g(converting m to kg or g)
Volume of liquid = 6.2 x 4.5
= 27.9cm3
Density = Mass/Volume = 4.9/27.9g/cm3 = 1.760kg/m3

14. 2005 Q3 P1
Water in A expands reducing/lowers density
This reduces/lowers up-thrust on block causing tipping to side A

15. 2006 Q12 P1


As the temperature changes the volumes of the gases in the balloons
change differently. The change in volume and hence the change in
upthrust will differ.
(2 marks)
16. 2006 Q17 P1
(a) A body fully or partially immersed in a fluid experiences an upthrust
equal to the weight of the fluid displaced (1 mark)

(b) (i)

Weight in air Weight in Weight in


water spirit

(ii) 100g: Uw = 0.12N Us = 0.09N


150g: Uw = 0.18N Us = 0.14N
200g: UW = 0.24N Us=0.18N (2 marks)

(ii) Relative density = upthrust in spirit


Upthrust in water
= average 0.09 0.14 0.18
0.12, 0.18, 0.24
= 0.76 (3 marks)
(c) Weight of air displaced= ρVg
1.25 x 1.2 x 10N
=15N;
= upthrust

Weight of helium = ρVg


0.18 x 1.2 x 10N
= 2.18N;

Weight of fabric = 3N
Forces downwards = 2.16 + 3 = 5.16N;
106
Tension = 15 – 5.16
= 9.84 N (4 marks)

17. 2007 Q19 P1

19 (a) A floating body displaces its own weight of the fluid on which it floats
(b)(i) To enable the hydrometer float upright / vertically 1 mark
(ii) Making the stem thinner/ narrower ( reject bulb) 1 mark
(iii) Float hydrometer on water and on liquid of known density in turn and 2 marks
marks levels; divide proportionally and extend on either side/ equal parts
(c)i) Tension; upthrust; weight 3 marks
(ii) As water is added, upthrust and tension increase; reaching maximum 3 marks
when cork is covered and staying constant then after weight remains
unchanged as water is added 11marks

18. 2008 Q14 P1


Heated water has lower density, hence lower up thrust

19. 2009 Q17 P1


a) A floating body displaces its own weight of the fluid on which it floats
b) I) w = T + U

ii) Vol = 0.3 x 0.2 x 0.2 m3


Weight = mg = 0.3 x 2 x 0.2 x 10500kg/m3 x 10
= 1260N

iii) Vol of liquid = vol of block


Weight of liquid displaced = Vpg
0.3 x 0.2 x 0.2 x 1200 x 10N
=144N

iv) T=W–U
1260 – 144N
= 1116N

c) Weight of solid = Weight of kerosene displaced


= 800 x 10x10-6 x 10 = 0.08 N
Mass = 0.008kg
Vol = 50cm3 density = m = 0.008
V 50 x 106 m3
= 160Kg/m3
20. 2010 Q10 P1
i) The upthrust in the liquid
u - 5.0 - 4.0 (working must be shown)
u = 0.96N

b) The volume of the solid.


Weight of liquid displaced = 0.96N
Mass of liquid displaced = 0.096kg
V = M/P = 0.096/800 = 1.2 x 10-1m3

107
1.2 x 102cm3
120cm3
21. 2010 Q18 P1
a) V = m/p or V = 4/3000
V = 1.33 x 10-3m3
(at least 2 dec places)
b) Upthrust = weight of liquid disp = vpg upthrust = weight of liquid displaced
= vpg
= 800 x 1.33 x 10-3 x 10 = 1000 x 1.33 x 10-3 x 10
= 10.64N = 13.33N

c) Weight of stone air = 4 x 10 = 40N


Reading of spring balance = 40 - 10 64 = 29.36N
40 - 13.33 = 26.67N
d) 85 - 10.64 = 74.36N or 85 - 13.33 = 71.76N

22. 2011 Q19 a, b, e P1


(a) Its weight is equal to the fluid it displaces
(b) Ship is hallow hence able to displace a lot of water equivalent to its weight
Filling water in the ballast tanks make the submarine heavier hence sinks.
Pumping air into the ballast tanks made the submarine lighter hence it floats
(e) (i) u =W1-W2=0.60-028 = 032N
weight of substance 0.08
(ii) Relative density = = =0.25
weight of equal volume of water 032
(d) To sink, water is allowed into ballast tanks and to float, pumps are used to
expel water from ballast tanks.
23. 2012 Q6 P1
 Oil film spreads over a large surface of the sea reducing inflow of air / oxygen
needed by the aquatic life
 Causes death of aquatic animals and plants / suffocation reduces sunlight
entering the sea.
 Beaches become dirty / causes pollution (of water)
 Poisons marine animals
 Contaminates sea water when taken in.

24. 2012 Q6 P1
When upthrust is equal to the weight of the balloon(and its contents)

25. 2012 Q18b P1


m
(i) volume of log =
e
20
=
800
= 2.5 x 10-2m3

(ii) v = weight of water displaced = evg or density x volume x gravity


20
= x 1000 x 10
800
= 250N
(iii) Tension = U – mg
108
= 250 – 200

= 50N;
26. 2013 Q18b P1
(a) (i) (I) the reading of the spring balance decreases
(II) the reading on the weighing balance increases

(ii) This is due to upthrust acting on the metal block placed in water
(iii) (I) reading = 2.46N + 2.80N = 5.26 N
(II) upthrust = 2.7 - 2.46 = 0.24 N = weight of displaced water

Volume of water displaced - volume of block = 0.024 = 0.000024m3


1000
0.27
Density = =11250 kg/m 3
0.000024

b) Increase in temperature leads to a decrease in density hence the hydrometer


skills deeper
27. 2014 Q19 P1

(a) (i) (I)


V=AXL
2x5(show workings)
=10cm3
(II)
Mass =Vol x density w = mg
= 10cm2 x 1 g/cm3 = 0.01 x 10 N/Kg -1
= 10 g or 0.1 kg = 0.1N
(ii)
Combined weight = upthrust
= 0.1 N
(iii) egh1=egh2
800 x h1 x 10 = 1000 x 10 x 0.05
h1=10 x 100 x 5 = 0.0625m
8000
28. 2015 Q18b P1
The volume of water icreases hence the water becomes less dense and the
block sinks further since the upthrust reduces
ELECTROMAGNETIC SPECTRUM
MARKING SCHEME

1. 1995 Q26 P1

Type of radiation Detector Uses


Ultra violet Photographic paper Cause ionization kills bacteria
fluorescence material OR operating photosular cells
photography
Infrared Phototransistor blackened Warmth sensation
thermometer
Radio waves Radio receiver or TV Communication
109
receiver

2. 1996 Q2 P1
Frequency: OR wavelength or energy

3. 1998 Q20 P1
Heating/ cooking/communication/eye/photographic film or plate/LDR/photocell.

4. 1999 Q28 P1
Radio waves, infrared, x-rays, Gamma rays.

5. 2000 Q29 P1
Microwave / cooker/ telephone/ radar etc

6. 2000 Q34 P1
Difference in energy of the state/ nature of atoms

7. 2001 Q27 P1
Longer radio waves are easily diffracted around hills/ radio waves undergo
diffraction easily.

8. 2001 Q4b P2
Energy released E = Ef – Ei = 5.44 x 10-19j = 4.08-19j
E = hf = hC

 = 6.63 x 10 x 3.0 x 108m
-34

4.08 x 10-19

= 4.88 x 10-7 m (4.87 – 4.90)


9. 2002 Q16 P1
Ultraviolet

10. 2003 Q27 P1


Fringes will be closer together / more fringes of violet light has a shorter wavelength
Red light has longer wavelength.

11. 2004 Q28 P1


X-rays Gama Rays
-produced by fast moving electrons -As a result of disintegration of nucleus
-Produced due to energy changes in
Level of atoms -due to energy changes with
nucleus
Of atoms
-Produced when energy changes in
Electronic structure of atoms -produced due to change in nucleus
Of atoms.

(Any one comparison give 1mk)

12. 2005 Q31 P1


In excited state the electron is in a higher (outer) energy level. As it falls back it
releases energy and may fall in steps releasing different energies (radiations)
(proton)

110
packets energy.

13. 2011 Q12 P2


Radio waves microwaves yellow light gamma rays

14. 2013 Q12 P2


Gas is apoor conductor of heat but radiant heat from the sun can penetrate it since it
has a shorter wavelength than that of the burning wood

15. 2014 Q4 P2

V=f𝝀
𝝀=3.0 x108
4 x 106

=75m.

ELECTROMAGNETIC INDUCTION
MARKING SCHEME

1. 1995 Q5 P2
(a) (i) The magnitude of the induced e.m.f is directly proportional to the rate at
which the conductor cuts the magnetic field lines
The induced current flows in such a direction as to oppose the changes producing
it.
(ii) Plugging a magnetic into a coil
 in speed its g twins as straight of magnetic field
111
 Results in an increased in the induced e.m.f

(b) (i) Energy is neither created nor destroyed


Make power constant
VU = Joules ( ½ ) current = charge ( ½ )
Count time
P = IV
For large V, 1 must lower for power input to be equal to power output
(ii) Vs - Vp OR Vs - Na
Ns Vp Vp NP
Ns = Vs x Np = 9 x 480
Vp 240 Ns = 18

2. 1996 Q31 P1
Eddy current

3. 1997 Q19 P1
mgh = ½ mv2 OR V2 = U2 + 2 as;
h=½ S = V2 = 36
= 18m; 2as 2(10)
S = ut + ½ at2 = 1.8m;

4. 1997 Q39 P1
By using laminated core

5. 1998 Q8 P1 (1 mark)
IP =N3 = Np= 20000 x 3 = 2000
Is=NP 30

6. 1998 Q13 P1
High resistance voltmeter takes less current/low current recording low current.

7. 2000 Q32 P1
Number of turns/ strength of magnetic field

8. 2000 Q33 P1
To reduce eddy currents in the armature

9. 2000 Q4 P2
(a) (i) Easily magnetized and demagnetized

(ii) Vp = Np 240 = 500


Vs Ns Vs 50
Vs = 24; V= VPR
VQP = 1/3 ; VPR = 8 V

10. 2001 Q31 P1

112
11. 2002 Q33 P1
Downwards

12. 2003 Q32 P1


Galvanometer deflects; Changing flux produced in p is linked to Q causing an
e.m.f to be induced / by mutual inductance an emf / current is induced in Q.

13. 2003 Q33 P1


Maximum deflection of G will be double; flux linkage doubles when the turns are
doubled.

14. 2004 Q5 P2
a) i) -Increasing me of turns/coils
-Increasing speed (rate) of rotation

b) In a motion produces Eddy currents. These cause force to act on plate causing
damping in B Eddy currents are reduced by slots
c) Rms = V peak/2
V peak = 12x14142=16.97v=17v

15. 2005 Q12, 13 P1

12.

13. to make the rotation continuous by changing the direction in the coil every
half cycle/turn also accept changing direction of the current every half
cycle/turn/maintaining the direction of current in field.

16. 2005 Q39 P1


When the magnet is moving into the coil there is change in magnetic field flux
hence an e.m.f is induced causing the galvanometer to deflect. When the magnet
stops inside the coil there is no change in magnetic field flux hence no e.m.f is
induced causing the pointer to go back to zero

113
17. 2006 Q20 P1
a) (i) P-supprisings
Q – Carbon brushes
(ii) rate of change of magnetic fluid changes

18. 2007 Q17 P2


(a) The induced current flows in such a direction that its magnetic effect
oppose the change producing it.
(b) As the diaphragm vibrates, it causes the oil to move back and forth in the
magnetic cutting the field lines, this causing a varying e.m.f to be induced in
the coil which causes a varying current to flow. (1 mark)
(ii) Increasing number of turns in the coil – increasing of the coil
Increasing the strength of the magnet ( any two correct) (2 marks)

c) Vp = Np
Vs Ns

400 = 1200
Vs 120
Vs = 40V

(ii) Ip = 600/400 = 1.5A (2 marks)


(iii) Ps = Pp = 600W
ls = 600/40 = 15A (1 mark)

19. 2008 Q18 P2


(a) Flux growing/ linking
No flux change
Flux collapsing

Switch closed: Flux in the coil grows and links the other coil inducing an
E.M.F
Current steady: No flux change hence induced E.M.F
Switch opened: Flux collapses in the R.H.S coil inducing current in opposite
direction

(b) (i) Reduces losses due to hystesis ( or magnetic losses)


Because the domain in soft- iron respond quickly to change in
magnetic (or have low reluctance) i.e easily magnetized and
demagnetized.
(ii) Reduces losses due to eddy current
Because laminating cuts off the loops of each current
Reducing them considerably

(c) (i) VP = NP P = IsVs


Vs Ns Is = 800
40
400 = 200
Vs 200
Vs = 40 Volts = 20A
114
(ii) Pp Ps
800 = 400 Ip
Ip = 800
400
= 2A

20. 2009 Q10 P2


(i) Movement of magnet causes flux linkage to change
E.M.F is produced in the cell.

When I flow from Q to P, a N pole is created which opposes the approaching


pole (longs law)
21. 2010 Q6 P2
High voltages imply low current so reduces heat /power losses

22. 2010 Q11 P2


Motion is oot of paper or moves or moves up

23. 2011 Q17 P2


(a) Phenomena of producing an emf in a conduct in a changing magnetic field
(b) (i) P = carbon or graphite brushes
(ii) Q Slip rings

(iii) • Increasing number of turns or coils


• Increasing speed of rotation
• Increasing strength of magnetic field
• Winding the coil on soft iron core
• Increasing cross-sectional area of the coil

b) (i) Vs = IR = 0.5 x 200 = 100V

c)

(ii) VP = np Vp = np x Vs = 1 x 100V = 10V


VS np ns 10
(iii) IP = ISVS = 0.5 x 100 = 5A

VP 10
24. 2012 Q16 P2
(a) i) The galvanometer will be deflected to one side and then back to zero, (ii) A
greater deflection will be obtained in the opposite direction as curreru
takes less time to die off than to build up.

(b) (i) The changing current in the primary coil induces a current in the secondi -
coil due to changing magnetic field of the primary current.
V s Ns 240 ×200
(ii) =
V Pp N p
→V s=
1000
=48V

115
power output 48× 0.8
(iii) Efficiency = ×100= ×100=80 %
power input 240× 0.2
25. 2013 Q8 P2
Ns V s 12 ×5
= →V s= =6V
Np Vp 10

26. 2014 Q15 P2


(a) (i) Resistance in the core
(ii) Using thick copper wires

(b)
(i)
NP VP NS 12 1
= OR = =
NS VS NP 240 20

240 20
= OR 20: 1 NS: NP = 20: 1
12 1

(ii)
P = VPIP
=240 x 0.36
86.4 W
(iii)
80 W

(iv)
eff = P Output or Eff – Part x q
P. Import p up
80 x 100 80 x w
86.4 56.4
92.6% 92.59%

27. 2016 Q15 P2


(b) i) The galvanometer will be deflected to one side and then back to zero, (ii) A
greater deflection will be obtained in the opposite direction as curreru
takes less time to die off than to build up.

(c) (i) The changing current in the primary coil induces a current in the secondi -
coil due to changing magnetic field of the primary current.
V s Ns 240 ×200
(iv) =
V Pp N p
→V s=
1000
=48V

power output 48× 0.8


(v) Efficiency = ×100= ×100=80 %
power input 240× 0.2

116
MAINS ELECTRICITY
1. 1997 Q24 P1
Extra heat is required to change ice to water / latent heat of fusion

2. 1999 Q34 P1
A-earth wire B – live wire C neutral wire

3. 1999 Q5(b) P1
2 2
V 240
(i) P= = =576W
R 100
P 576
(ii) P=VI =¿ I= = =2.4 A
V 240
(iii) heat lost by heater = heat gained by water
Pt =mc ∆ θ
0.2× 4200 × ( 95−23 )
t= =34.56
576
576× 2× 30
(iv) number of KWH = =34.56
576
Coast = 34.56 x 5 = 172.80

4. 2001 Q32 P1
E = pt = 60 x 30 x 60 x 60J OR E = 60/1000 kW x 36hrs
In kWh = 60 x 36 +60 x 60 J E = 0.06 x 36
1000 x 60 x 60
= 2.16 Wkh E = 2.16kWh

5. 2002 Q18 P1

E = V2 t = 2402 x 120 = 14400J


R 480

6. 2002 Q30 P1

I = PT = 300 + 750 + 2000 = 12.2A Suitable fuse = 15A


V 250

7. 2003 Q17 P1 `

P = V2 / R P = VI = I2R
75 = 240 x 240 or Do not accept p = VI alone without I2R
R R = p/12
= 768  R = 75 x 240 / 75 x 240 / 75 = 168 

8. 2004 Q20 P1
Either p=VI = V2/r
When V reduces power reduces
So rate of heating reduces
Or V=IR
P=I2 R (reducing IR reduces power so rate of heating reduces.

9. 2005 Q19 P1
2
(i) = V /-R

117
2500= 240 2/R
R=23.04 or (23.03)

(ii) P=IV
I P/V = 2500/240 =10.417A
V = V/I= 240/2500
2500
= 23.04R (23.03)

(iii) P= IV and V=IR or I2 R


R= 240 x 240
2500
R= 23.04R

10. 2006 Q21 P2


(a) (i) P = Ring circuit
X = Neutral ( point or terminal)
Y = Live ( point or terminal)

(ii) I Purpose of R – or fuse; is a safety element in a circuit


against excess current
II R is connected to Y but not X to ensure that when it breaks
a circuit any gadget/ appliance connected does not remain live.

(iii) Earthing is necessary in such a circuit to guard against electric


shocks.

(b) Cost of electricity


1.5 kw x 30h x 8 Kshs = Kshs 360/=

11. 2010 Q6 P2
Implies low current So reduces heat losses / power loss or I2R loss reduced
P = I2R should be accompanied by power loss
NB: Heat losses / power loss

12. 2011 Q13 P2


Reduce power lost during transmission by reducing current

13. 2012 Q12 P2


alternating voltage can be stepped up, or enhances reduced power losses;
alternating current can be stepped down, or enhances reduced power losses;
(step up or down a.cc) (1 mark)

14. 2012 Q16 P2


Total power covered = 100 + (2 x 60) + 30 + 250) watts
250
Consumption in Kwh = ×70=17.5Kwh
1000
250
Cost of electricity = 17.5 x = Ksh 7.00
1000

15. 2012 Q16 P2

118
(a) (i) Pointer deflects up to a certain; maximum value and then return to zero;
(pointer shows a momentarily deflects (2 marks)
(ii) There is a deflection in the opposite direction then back to zero; As Flux
in A falls, flux in B also fails and causes induced e.m.f in the opposite
directions

(b) (i) Flux / current in the primary is constantly changing its direction/ magnitude
that so that the resulting flux (which link coils) is constantly changing its
direction. Therefore alternating e.m.f is induced in the secondary coil;

Vs Ns
(ii) =
Vp Np
Vs 200
=
240 1000
Vs = 48V
Power output
Efficiency = ×100 %
Power input
IsVs
= IpVp ×100 %

0.8 × 48
= 0.2× 240 ×100 %

= 80% or 0.8

16. 2013 Q9 P2
So that they can operate at the same main voltage
So that they can be operated independently

17. 2015 Q13 P2


VS = 24V, Power input = 200W
Power output = 200W (100% efficient)
P = VI
200W = 24 X 1
I = 8.33A

18. 2015 Q15b-c P2


(b) It has a thin short piece of wire which overheats and melts if a current
greater than its rating flows through it hence breaking the circuit that
it connects
P 2500 W
(c) (i) I = = =10.41 A
V 240 W
fuse is not suitable because it exceeds the fuse rating

(ii) no of kWh = 2.5 x 3 = 7.5kWh


Cost = 7.5 x 0.8
= Kshs. 6.0

119
CATHODE RAYS AND CATHODE RAY TUBE
MARKING SCHEME

1. 1996 Q32 P1
Low negative voltage is applied on control grid, which control the number of
electrons reaching the screen (1 mark)

2. 1998 Q19 P1
In TV (CRT) deflection is by magnetic field, while in CRO deflection is by
electric field. X-Y plates.

3. 2000 Q35 P1
ATV (CRT)has two time bases while a CRO has only one.
In CRT it produced 625 lines per second while CRO is 25 lines per second.
- C.RT (T.V) use magnetic field to deflect electrons while C.R.O use electric
field

4. 2001 Q31 P1

5. 2001 Q4a P2
i) Filament heats up cathodes; causing electrons to boil off the cathode.
ii) Grid controls brightness of spot since it is negatively charged it repels
the electrons reducing number of electrons
iii) A vertical line would appear/spot oscillates vertically
iv) Deflection in TV is by magnetic fields.
v) Magnetic field produces greater deflection on electrons beam allowing
wider screen.

6. 2002 Q29 P1

120
7. 2003 Q34 P1

8. 2004 Q32 P1

Check-at least Three complete troughs/Crest


- Amplitude range 6.5 squares

8.5 squares

9. 2006 Q12 P2
Straight beam from gun to screen OR no gravitational effect on the beam.

10. 2006 Q13 P2


Tube

Magnetic field

Electron path

11. 2007 Q18 P2


(a) (i) A Grid
B Filament
(ii) Filament heats cathode
Electron boil off cathode ( theremionic emission)
(iii) Accelerating electrons
Focusing electrons
(iv) Across X - plates
(v) To reduce collisions with air molecules that could lead to ionization

(b) Height = 4 cm
Peak value =4x5
121
= 20V

(ii) 2 wavelength = 16 cm
T = 8 x 20 x 10-3
= 0.16S
l 1
f = /T = /0.16
= 6.25Hz

12. 2008 Q12 P2


Spot moves up and down

13. 2011 Q18a P2


- Cathode rays are deflected by magnetic field while electromagnetic
radiations cannot.
- Cathode ray have lower speed than electromagnetic radiations

14. 2012 Q18a P2


(a) • cathode rays have charge but electromagnetic radiations do not have charge
 cathode rays are particles and have a mass while electromagnetic radiation are
waves.
 cathode rays travel at a speed depending on accelerating voltage but
electromagnetic radiation travel at a speed of light in a vacuum.

(b) (i) M = grid, N = accelerating anode

(ii)The cathode is heated by filament and electrons are released from cathode
by thermionic emission.
(ii) (I) across Y plates (II)
across C plates

(iii) This is to reduce collision, (hence ionization) with air molecules in the tube.

(c) (i) V = 5 x 2 = 10V


(ii)

15. 2013 Q18a P2


(a) (i) Thermionic emission i.e. the filament heats up the cathode
(ii)A spot is seen on the screen
(iii) (I) to focus the electrons towards the screen
122
(II) to accelerate the electrons towards the screen

(b) (i) 2x2 = 4 milliseconds for one wave 4


4
T= =0.004 s
1000
1 1
(ii) f = = =250 Hz
T 0.004

16. 2014 Q13 P2


Q = It
Q
n= C
=20 x 104
1.6 x 10-19
=1.25 x 1015 (electrons optimal e is not a unit)

17. 2015Q12 P2

123
X-RAYS
MARKING SCHEME

1. 1995 Q28 P1
– Lead - Very dense/ has high atomic mass

2. 1998 Q4 P1
Detecting imperfection in metal structures/block/flaws

3. 1999 Q23 P1
Kinetic energy ray / heat energy. Electromagnetic

4. 2000 Q35 P1
X – rays produces - Hard X – rays are produced
Higher E.H.T results in faster electrons hence higher energy X- rays

5. 2001 Q33 P1
Pd across Anode – cathode Or anode potential (voltage)

6. 2002 Q32 P1
Penetrating power (high)

7. 2003 Q25 P1
X – rays produces - Hard X – rays are produced
To with stand high temperature / high melting point.

8. 2004 Q28 P1
X-rays are outside the nucleus while Gamma rays are from inside the nucleus

9. 2004 Q33 P1
X-rays (Hard) Soft-rays
-Shorter Wavelength -Longer wavelength
-More energetic -Less penetrating
-High Frequency -Low frequency
-Produced by high voltage -Produced by low voltage.
124
-Produced by fast moving electrons -Produced by slow moving e
-electrons
10. 2005 Q33 P1
To withstand the high temperature (immerse heat) prevent the target from melting
due to high temperature or immense heat.

11. 2006 Q14 P2


Resulting X- rays have shorter wave length/ hard/ high frequency because electrons
have higher K.E (2 marks)

12. 2008 Q13 P2


Frequency increases
Accept Becomes hard
Wavelength decreases
Strength / quality

13. 2008 Q19 P2


(a) (i) Hard X – Rays
(ii) They are more penetrating or energetic

(b) (i) A cathode rays/ electrons/ electron beam


B Anode/ copper Anode

(ii) Change in P.d across PQ cause change in filament current


OR temperature of cathode increases
This changes the number of electrons released by the cathode hence
intensity of X- rays

(iii) Most of K.E is converted to heat

(iv) High density

(c) Energy of electrons is = QV= ev


= 1.6 x 10-19 x 12000

Energy of X- rays = Hf
= 6.62 x 10-34 x f
6.62 x 10-34 x f = 1.6 x 10-19 x 12000
F = 1.6 x 10- 19 x 12000
6.02 x 10-3f
= 2.9 x 1018Hz
Accept ev = Gf
F = ev/g

14. 2009 Q11 P2


Increase in p d increases I in filament OR. Increases heating effect. This produces
more electrons by thermionic emission. Hence results on more intense X-rays.
125
15. 2010 Q12 P2
Increasing the accelerating voltage, or increases in p.d increasing heating effect

16. 2012 Q18a, b


(a) (i) To produce electrons; by thermionic emission; (2 marks)

(ii) To accelerate the electrons to give them enough K.E. to produce X-rays at the
Anode (2 marks)
(iii) To absorb stray X-rays, thus protecting the operator from those rays; (1 mark)

(b) Increases K.E. of electrons and hence causes X-rays of higher frequency; (1 mark)
OR
- X - ray are more penetrative
- X - rays of shorter wavelength.

17. 2013 Q10 P2


X-rays ionizes the air between the plates. The positive ions move towards
the negative plate and the negative ions towards the positive plate. A
potential difference is therefore created resulting to a flow of current.

18. 2014 Q11 P2


It stops fast moving electrons whose kinetic energy is concerted to heat/energy of
electrons are converted into heat

17. 2013 Q10 P2


X-rays ionizes the air between the plates. The positive ions move towards
the negative plate and the negative ions towards the positive plate. A
potential difference is therefore created resulting to a flow of current.

126
PHOTO ELECTRIC EFFECT
MARKING SCHEME

1. 1995 Q27 P1
E2 = E1 + h f i or E2 – E1= h = c/l
h= plank constant
c- Velocity of light
l- Wave length of light

2. 1995 Q29 P1
Extrapolation on graph (line to touch frequency)
Reading on graph to (4.0 + - 0.2) x 1014Hz

3. 1996 Q7a,b P2
( a)

Close switch S
Vary pd until G deflects
(b) l)

127
Finding f
See graph
Axes labeled
Scale
Pointed plotted
Straight line

(ii) Work function Φ is given by Φ hf0


F0 is the x – intercept of graph
F0 ( from graph) = 1.2 x 1015 HE
Φ = 6.63 x 10-340.5 x 1.2 x 1015
= 7.96 x 10-19 J

4. 1997 Q5(c) P2
c) HF = hfo + ½ mv2
= (3.2 + 82 ) x 10-19 = 11.2 x 10-19 f= 11.2 x 10-19
6.63 x 10-19
l= c = 3.0 x 108 x 6.63 x 10-34 = 1.76 x 10m
F 11.2 x 10-9
5. 1998 Q4(b) P2
(i)

ii) When light rays strike cathode C surface electrons gain photon (energy)
hence the cathode.
iii) Draw a simple circuit including the photocell to show the direction of flow
of current.

128
(iv) E = hf
= 6.63 x 10-34 x 8.6 x 10-14J
= 5.70 x 10-19 J

6. 1999 Q30 P1
Ultra violet releases electrons from zinc plate by thermal emission.
On removal of electrons, zinc becomes +vely charged.
Positive charge on zinc discharges/ neutralizes the charged on the electroscope.

7. 1999 Q7 P2
(i)Change V using the variable resistance and record V and correspond values
of I as given by Q

(ii)

h h h
(b) (i) from eVco = f + f 0 but = gradient h = gradient x e
e e e

0.40
Gradient = 14
=4.0 ×10−15
( 5.5−4.5 ) × 10
h = 1.6 x 10-19 x 4.0 x -15
= 6.40 x 10-34Js
(ii) work function, ∅ = hfi
By extrapolating the graph to cut the x- axis, read off, f0 = 4.5 x 1014 Hz
∅ 6.4 x 10-34 x 4.5 x 1014
= 2.88 x 10-19J

8. 2000 Q30 P1
Difference in energy of the state/ nature of atoms

9. 2001 Q34 P1
Energy released rE = Ef – Ei = 5.44 x 10-19j = 4.08-19j
rE = hf = h C
l
l = 6.63 x 10-34 x 3.0 x 108m
4.08 x 10-19

= 4.88 x 10-7 m (4.87 – 4.90)


10. 2002 Q34 P1

129
Work function of metal / min energy required to eject e-1 for excess energy work
function

11. 2002 Q7 P2
a i) mV light removes electrons on zinc plate. This lowers the excess charge
constant (negative) on leaf leading to collapse/ becomes less negative (more
positive)
ii) Since mv light removes electrons positive charge re attracts the electrons
thus keeps the charge constant and so leaf does not collapse.
bi) Frequency of incident light / energy of proton / energy of light work
function of surface
ii) From Kemax = hf - q
h is slope of graph
Slope = (10 – 20) x 10-19
(2.6 – 1.4) x 1015
H = 6.7 x 10-34 fs
At Kemax = q hf = 0
Extrapolation shown or
Read off fo = 1.07 x 1015 Hz
Q =1.07x 1015x 6.67 x 10-34
= 7.4 x 10-19

c) Kemax = hf q
= 6.67 x 1034 x 5.5 x 1014
1.6 x 10-19
= 2.29 eV

Since hf< q no photo elective effect


E= hf = 6.67 x 10-34 x 5.5 x 1014
Or q = 2.5 x 1.6 x 10-19

12. 2003 Q35 P1


Q = hf0 =Wo or & = hfco
= 6.63 x 10-34 x 9.06 x 1014j
= 6.01 x 10-19 J or 6.0061 x 10-10 or 6.0 x 10-19 if working is shown.

13. 2004 Q34 P1


hf0=ωc=
Fo = ωc/h=32x16 x10-6.62x10-34
= 7.73 x 1014 H2 or 7.732 x 1014 H2 or 7.734 x 1014 H2
=7.73 x 1014H2 or 7.732 x 1014H2 or 7.734 x 1014H2

14. 2004 Q7 P2
i) Photoelectric effect- is the emission of electrons from a surface when radiated
with radiations of sufficient frequency.
Correct circuit must work i.e cathode connected to (-ve) Emphasize on mA
cell connected and v in parallel

ii) Slope = 1.28-0.10/(7.7-4.8) x 10 14


h= Slope x e
= 1.18 x 1.6x 10-19/29 x1014
= 6.51 x 10-34JS
(5.82 – 6.66) x 10-34 JSAlt – Selecting 2 pts from graph
- Substitution in simultaneous equs

130
-Value of h
-Value of 
Fs (Threshold Frequency) = 4.55 x1014 (where graph cuts the axis)
Range (4.4 – 4.6) x 2014
Work function  = 6.51 x 10 -34 x 4.55 x 10 14
= 2.96x 10-19J
Range (2.56-3.06) x 10 -19J

c) ½ mv2 max = hf-


hf= 6.51 x 10x 3x1015
KE max = 1.953 x 10-18 – 6.4 x 10 -19
= 1.31 x 10-18
Range (1.12 – 1.31) x 10 -18J

15. 2005 Q31 P1


In excited state the electron is in a higher (outer) energy level. As it falls back it
releases energy and may fall in steps releasing different energies (radiations)
(proton) packets energy.

16. 2006 Q10 P2


work function – minimum energy required to eject an electron from a Metal

17. 2006 Q11 P2


Electrons do not move

18. 2007 Q11 P2

19. 2007 Q12 P2


The higher the intensity implies greater number of electrons and hence higher
saturation current

20. 2009 Q18 P2


a) Electrons are emitted from Zn plate
Reduced of charge on the leaf.

ii) Any electron emitted is attracted back to the electroscope


iii) Photons of infra red have to lower f than U - V have less energy to eject to the
electrons

131
b)i} Number of electrons emitted will increase
{ii) Max K.E of the emitted electrons will increase

c)i) v = λ x f 0
f 0 3.0 x10 8
8.0x 10 -7
= 3.75 x 1014 Hz
ii.} W= hf 0
=6.63xl034 x 3.75x1O 14
=2.49 x 1Q19 J .= 1. 55 eV
1.6x 10 -19

Iii} K E max =hf-hf o


=h (8.5-3.75)x1O14
6.63x4.75x1O14
3.149 x10 -19 Joules
= 1.96828Ev
21. 2010 Q18 P2
(a) the intensity of radiation
(b) (i) stopping potential is negative potential sufficient to just stop the movement of
electrons
h 3−0
(ii) (I) gradient = = =3.95× 10−15
e ( 12−4.4 ) × 1014 Hz
H = 3.95 x 10-15 x 16 x 10-19 = 6.32 x 10-34 Js

−ω0
(II) y-intercept =
e
−ω0
= -1.75V ω 0= 1.75eV
e
22. 2011 Q14 P2
Threshold frequency minimum frequency of a radiations required to remove
an electron from a matal surface

23. 2012 Q18b P2


(c) E = hf;
= 6.63 x l0-34 x 7.5 x l014
-4.97 x 10-19J or 9.7 x 10-20J

K.E = 4.97 x 10-19 4.0 x 10-19


= 0.97 x 10-19J; or 9.7 x 10-20J (4 marks)

24. 2013 Q12 P2


Kinetic energy = E – T

25. 2014 Q6 P2
Electrons absorb enough energy
Electrons are affected from zinc plate
Learning electroscope positively charges the leaf is repelled by the stem

26. 2015 Q 14 P2
(a) Type of metal
Frequency of radiation / energy of radiation
132
Intensity of radiation

c 6.63 ×10−34 −19


(b) (i) E=h = =4.626 ×10 J
λ 4.3 ×10 −7

(ii) Potassium. This is because the work function of potassium is lower than the
energy of incident radiation

(iii) E = w0 + K.E K.E = 4.63 x 10-19J – 3.68 x 10-19J = 9.46 x 10-20J = ½ me V2


9.46 x 10-20J = ½ x 9.1 x 10-31 x V2
V = √(2.087 x 1011) = 4.569 x 105m/s

RADIOACTIVITY
MARKING SCHEME

1. 1995 Q32 P1
X = 14 Y = -1

2. 1996 Q32 P1
Low speed / high charge / more massive/ size is large/ bigger` ( 1 mk)

3. 1997 Q34 P1

4. 1997 Q35 P1
133
No of t1/2 = 2
No of atoms remaining
= No = 5.12 x 1020
2n 22
= 128 x 1020 atoms
Atoms decayed =
(5.12 – 1.28) x 1020
= 3.84 x 1020 atoms

5. 1998 Q12 P1
50 = (I)n n =3(half-lives)
400 (2)n
Half –life 72 = 24 min.

6. 1999 Q7 P1
Particles are + vely charged, if majority deflected most atom is empty.
Deflection  existence of a +vely charged nucleus.
Few deflected  nucleus is small/mass is concentrated at the centre

7. 1999 Q35 P1
Z Y Z  Z+1 +o – 1e
Or Atomic number charges by / New is a head of the old or Z + 1

8. 1999 Q6(c) P1
c i) Time taken for half of the radio acute material to disintegrate.
ii) Correct readings for 60 and 30 time 25 + 2 minutes

9. 2000 Q36 P1
From 300 – 150 = 74 S 200 – 100 = 76 S
Average = 75 ± 1 other values on the graph could be
used
Donor impurity is the atom introduced into the semiconductor (doping) to provide
an extra electron for conduction.

10. 2001 Q34 P1


r -  (Beta)or ie B = 82 C = 206

11. 2002 Q27 P1


No of t ½ =21 = 4
5.25
Fraction remaining = 1/24 = 1/16

12. 2001 Q31 P1


107 – 42 = 65

13. 2002 Q2 P2
 - eta radiation
Force is of the circle implying negatively charged (Fleming’s left hand rule)

(b i) K= alpha (ii) X= 88 Y= 288

c i) Increase in thickness

134
(ii) Increase in thickness reduces the radiation reaching the Geiger tube
(iii) Increase in pressure
(iv) Increase roller pressure squeezes metal sheet (possess more) reducing the
thickness of foil coming out of them.
(v) Alpha particles have little penetration very few or none pass though foil.

14. 2003 Q18 P1


Beta particle  (Do not award for beta ray) Beta radiation Beta emission

15. 2003 Q4(b) P1


(i) To allow all radiations to penetrate
i. On entering the tube, the radiation ionizes argon gas. The positive ions flows
towards the cathode and negative ions towards the anode. This creates potential
difference that results to flow of pulse current.

ii. to absorb the energy of positive ions before they cause secondary electron
emission.
16. 2004 Q3 P2
a) i) Produce alcohol vapour
Cools alcohol vapour below condensation temperature or cools air so that
alcohol vapour condenses.

ii) Radiation from source ionizes air along its path; alcohol condenses
around these ions; forming droplets or traces; nature of traces identifies
radiation.
iii) Can detect,__ While electroscope on , can identify nature of radiations, is more
sensitive.)

b) i)

ii) Delayed 1x1020___________ ½ x 1020 - ________ ¼ x 1020 _______ 1/8 x 1020


= 0.125 x 1020 = 1.25 x 1020
Left 1 x 1020 ----------- 1/8 = 0.875 x 1020
(Subtraction ) = 8.75 x 1019 Atoms.

17. 2005 Q36 P1


m- Alpha () particle/ radiation/decay
n- Beta ()
x- Polonium (Po)

18. 2005 Q7 P2
(a) Radioactivity is the spontaneous disintegration of unstable nuclei so as to
stabilize

135
When radiation enters via mica windows, the argon gas is ionized; the electrons
going to the anode and positive ions going to cathode; thus a discharge is
suddenly obtained ( PULSE) between anode and cathode and registered as a
particle by counter. The discharge persists for a short time due to the quenching
effect of halogen vapour.

(c) Half life average t ½ = 24.5 min (error transfer)

12 12 12
(d) t(min 40 28 16 4

Activity 480 960 1920 3840


3 half – lives
t = 4 min

19. 2006 Q15 P2


a = 234 + 4 = 238
b = 92 – 2 = 90

20. 2007 Q13 P2


a = 234
b= 82

21. 2008 Q14 P2


Beta particle
Gain of an electron OR
Mass number has not changed but atomic number has increased by 1
Atomic number has increased by one
Nature will not affect the speed

22. 2009 Q19 P2


a) β particle

b) i) ionizes attracted towards electrodes


Collusions with other molecules cause a avalanche of ions which on attraction
to the electrodes causes the discharge
ii) Ions are attracted towards electrodes

c ( i ) X =36
Y=92
ii) Small; decreases in mass
Loss of mass
Change in mass
Mass defect
iii) Each of the neutrons produced at each collision causes further collision' with
uranium atom causing chain reaction.
136
23. 2010 Q9 P2

24. 2011 Q19 P2


(a) Alpha very high ionizing effect

(b) No of t1/2 = 19.15 =5


3.83
A = 1.6 x 103 = 1.6 x 103 = 0.05 x 103 = 50dis/s
25 32
25. 2012 Q6 P2
β and γ rays

26. 2013 Q7 P2
(a) a = 1, b = 0 (b) neutron

27. 2014 Q9 P2
(a) X 222
(b) Y 86

28. 2015 Q16 P2


(a) alpha particles lose some of their energy as they travel because they cause
ionization of air particles on their path. Beta particles lose very little energy
since they cause less ionization
1 50% 2 25% 3 12.5%
(b) 100% → → →

(c) (i) the window allows radiation to enter the tube


(ii) bromine acts a quenching agent. It reduces or quenches the energy of
alpha particles

(d) (i) (I) Alpha particles rapidly cause eavy ionization rapidly losing energy hence
their short range
(II) alpha particles are massive and their pat cannot be changed by air molecules

(ii) - they can detect all types of radiation


- they are durable and portable
- they are relatively inexpensive

29. 2016 Q8 P2
a = 234
b= 82

30. 2016 Q17 P2


(a) Alpha very high ionizing effect
137
(b) No of t1/2 = 19.15 =5
3.83
A = 1.6 x 103 = 1.6 x 103 = 0.05 x 103 = 50dis/s
25 32

ELECTRONICS
MARKING SCHEME

1. 1995 Q33 P1
(a) Common or sillen ( semiconductor) is doped with impurity atoms which trivalent (
e.g boron or indium) intensity in currency on pole group 4 doped with trivalent

2. 1997 Q33 P1

138
3. 1997 Q6(a) P2
a i) Semiconductors – conducting is by holes Conductors – conducting is by
electrons
ii) Semiconductors – silicon, germanium Conductors – copper , tin iron.

4. 1998 Q21 P1
Diode is forward-biased, no current flows
Current flows when the switch is closed but when terminals are reversed, no current
flows

5. 2000 Q37 P1
Donor impurity is the atom introduced into the semiconductor(doping) to provide
an extra electron for conduction.

6. 2000 Q7(a) P1
Pure semi- conductors doped with impurity of group 3, combination creates a hole
( positive), this accepts electrons.

7. 2001 Q5a, b P2
(a)

8. 2003 Q19 P1

Dope with group III element (e.g. Boron, Al, Ga). Three silicon electrons pair
up with impurity atom electrons. One electron of silicon has no electron to pair
up; hence a hole is created(For correct structure without explanation but
showing a group three element.

9. 2005 Q35 P1

10. 2005 Q4 P2
(a) (i) Pure Silicon or germanium is doped with prevalent impurity i.e.
phosphorous.
(ii) Four of the fire valence are paired with semi- conductor electrons

139
(iii) The fifth electron is left unpaired and so conducts
NB; Doping pairing and conducting must be mentioned

(b) (i) In the first half – cycle A is a positive making D2 and D3 to be forward
biased, so current flows through D2 R and D3 to B.
In the second half – cycle, B is positive making D4 and D1 forward biased. The
current flows through D4 R and D1 to A

(ii)

(iiii) The capacitor is charged when p.d is rising and stores charge
It discharges through the resistor when p.d is falling
This makes output smooth i.e reduces humps
(c) hfe = ∆Ic
∆IB

120 = ∆Ic
20B/A
Therefore ∆Ic = 120 x 20 MA = 2.4mA

Output p.d charge = RL x ∆IC


1000R x 2.4 mA
= 2.3v
11. 2006 Q16 P2

12. 2006 Q7 P2
(a) Pure semi- conductors doped with impurity of group 3, combination creates a
hole ( positive), this accepts electrons.

(b i)

(i) At Ve E = 0
140
Vcc = Ic R L
Lc = 9/1.8 K W lc = 10
VeE = Vcc = 9

(ii) ∆lc = (see graph) = 3.5 – 1.2 = 2.3 mA


B= ∆lc
∆lc
2.40A
40 μ A
= 60.
13. 2007 Q14 P2

14. 2009 Q13 P2


Diode in 6{a}) is forward biased while in 6 (b) is reversed biased OR
Battery in 6(a) enhances flow of e. across the barriers while in 6(b) barriers
potential is increased.

15. 2010 Q14 P2

16. 2011 Q19(c, d & e) P2


c. Semi conductor in which impurities have been added to change conductivity
d. By connecting it in a forward biased mode.

e. (i)

(ii) Across QS

17. 2012 Q13 P2


141
18. 2013 Q13 P2
Four electrons from arsenic bond covalently with germanium leaving a free electron
that is responsible for conductivity.

19. 2014 Q8 P2

1 mark for correct bias


1 mark for both ammeter & voltage
1 mark for Rheostat varying p.d across diode
19. 2013 Q13 P2
Four electrons from arsenic bond covalently with germanium leaving a free electron
that is responsible for conductivity.

20. 2016 Q19(c, d & e) P2


f. Semi conductor in which impurities have been added to change conductivity
g. By connecting it in a forward biased mode.

h. (i)

(ii) Across QS

MIXED CONCEPTS
MARKING SCHEME

1. 1998 Q27 P1
Moon covers the sun/obstruction of sun by the moon
Both heat and light have same velocity/both are electromagnet waves.

2. 2000 Q5 P1

142
Since the system is in equilibrium let A be the area of piston and P the pressure of
steam
P x A x 15 = ω (15 + 45)
2.0 x 105 x 4 x 104 x 15 = W x 60
ω = 20N

3. 2005 Q3 P1

Water in A expands reducing/lowers density


This reduces/lowers up-thrust on block causing tipping to side A

4. 2007 Q14 P1
Since the quantity of water A is smaller, heat produces grater change of temperature
in A; This causes greater expansion causing the cork of temperature in A; this cause
greater expansion causing the cork to sink further.
Per unit volume/ greater decrease in density/ lower density in A

5. 2016 Q6 P1
Since the quantity of water A is smaller, heat produces grater change of temperature
in A; This causes greater expansion causing the cork of temperature in A; this cause
greater expansion causing the cork to sink further.
Per unit volume/ greater decrease in density/ lower density in A

143

You might also like